Peds Core Questions Review

Olmsted Syndrome
- inheritance and gene
- classic features
- other findings
- at risk for what within the keratoderma?
Click the card to flip 👆
1 / 163
Terms in this set (163)
is a rare PPK associated with mutilating PPK and periorificial keratotic plaques. AD inheritance with a mutation in transient receptor vanillin 3 gene (TRPV3)

Clinical features classically include well demarcated PPK with an erythematous border and periorificial keratotic plaques. The mutilating PPK may progress and lead to flexion contractors and even autoamputation. other findings include alopecia, subungual hyperkaeratosis and nail dystrophy, corneal dysplasia, deafness. patients are at higher risk for skin infections, SCC, and melanomas within areas of keratoderma
Vohwinkel syndrome typically presents with "starfish-shaped" hyperkeratotic plaques on the hands, feet, knees, and elbows and pseudoainhum. It does not classically present with periorificial involvement, and it is associated with GJB2 or LOR mutation.

Mal de Meleda typically presents with transgradient hyperkeratotic plaques on the palms and soles, as well as psoriasiform or lichenoid papules on the elbows and knees. Fissuring of skin and malodor are frequent complaints. It does not classically present with periorificial involvement, and is associated with SLURP1 mutation.

Papillon-Lefèvre syndrome is a rare PPK with destructive periodontal disease and premature tooth loss. It does not typically present with periorificial involvement, and is associated with CTSC mutation.

Pachyonychia Congenita may also show significant nail dystrophy, however does not typically present with periorificial involvement, and is due to mutations in KRT6, KRT16, and KRT17
A 1-month-old girl is brought to your clinic for a small bump on the glabella. Her mother reports that this has been there since birth and has not changed in size or color. It appears to be asymptomatic. On examination, there is a subcentimeter skin-colored firm nonmobile subcutaneous mass. What potential next step would most clearly evaluate the soft tissue abnormality?
MRI

Midline congenital masses may represent a developmental anomaly of the head and neck. Developmental anomalies result from faulty morphogenesis during embryogenesis. Midline developmental anomalies on the glabella/nasal region include dermoid cysts, nasal gliomas, and cephaloceles. A skin-colored firm mass is most likely a dermoid cyst Midline developmental anomalies have the potential for intracranial extension and must be evaluated with imaging prior to any procedures. MRI is the imaging of choice to evaluate for softtissue changes. Ultrasound has limited sensitivity (Option B.). Midline congenital masses with intracranial extension are at risk of CNS infections as well as underlying bony defects. Therefore, surgical excision (Option D.) is often the recommended treatment (after imaging evaluation), and pre-operative biopsies (Option E.) should be avoided to prevent ascending infections. Benign reassurance is not the correct answer (Option A.)
A 2-year-old male is brought in by his mother to your dermatology clinic for evaluation of lesions that have continued to come and go since about 6 months of age. His mother describes a "bumpy rash" mostly involving his scalp and face and noting the development of new spots in this region which then spontaneously resolve. Sometimes lesions develop into what the mother calls a "scab." She has not treated the eruption as she was told by her general pediatrician this was a common rash in babies that should self-resolve without treatment. He is an otherwise healthy child growing appropriately and meeting all developmental milestones. On examination, you observe scattered erythematous papules and pustules with a few areas of hemorrhagic crust mainly on the scalp and some on the face. Based on the history and exam findings, what is the most likely diagnosis and what findings would be expected on smear of pustular contents?
Eosinophilic pustular folliculitis of infancy (EPF); eosinophils on Wright stain. EPF is one of the more uncommon conditions within the category of vesiculopustular eruptions of newborns and infants. It shares some of the features seen in the more common infantile eruptions such as erythema toxicum neonatorum (ETN) and transient neonatal pustular melanosis (TNPM), but can be differentiated based on findings from history, exam, and smear of pustular contents. EPF is more common in males, presenting on average around 6 months of age and almost always before 15 months of age. Although it can be present at birth, this is not typical. Waxing and waning, recurrent papules, pustules, and vesicles are present mainly on the scalp and face and can be pruritic. These lesions resolve with hemorrhagic crust or even scars. This condition tends to continue for several months to years, usually self-resolving by 3 years of age. Smear with Wright stain of purulent material shows abundant eosinophils without bacteria, yeast, or viropathic changes. Histopathology shows an eosinophilic infiltrate around follicles, within the outer root sheath, or in the dermis. There is not always follicular involvement on pathologic evaluation, however.
Maculopapular cutaneous mastocytosis (MPCM), better known as urticaria pigmentosa is the most common type of cutaneous mastocytosis. Mastocytosis is a group of disorders identified by abnormal mast cell proliferation and accumulation.

Mastocytosis is typically subdivided based on the involved anatomical locations: 1. Cutaneous mastocytosis (CM) - limited to the skin 2. Systemic mastocytosis (SM) - involves multiple extracutaneous tissues, may have skin involvement but not required Within the pediatric population, the majority of cases present within the first year of life, with cutaneous involvement only The development of mastocytosis is related to dysregulation of the stem cell factor (SCF) (kit ligand). This growth factor is responsible for the normal development and propagation of mast cells. But, when gene mutations affect this pathway, (gene encoding KIT) mastocytosis may occur. (Bibi et al. Immunol Allergy Clin North Am. 2014) Cutaneous mastocytosis is classified according to consensus statement by the World Health Organization (WHO) (Valent et al. Blood 2017): Maculopapular cutaneous mastocytosis (MPCM) (urticaria pigmentosa) (UP) - most common, sub-types include the following: nodular form, plaque form, monomorphic (adults), polymorphic (children) Diffuse cutaneous mastocytosis (DCM) Localized mastocytoma of skin The most common symptoms seen with CM are pruritus and flushing secondary to mast cell mediated release of vasoactive mediators (histamine, leukotrienes, prostaglandins). Another classic finding in CM is Darier's sign (see image). Darier's sign is the development of erythema and pruritus following irritation of the skin infiltrated with mast cells. (Soter et al. Hemaol Oncol Clin North Am. 2000) MPCM most commonly presents with yellow-brown/red-brown macules or minimally raised papules, but nodules and plaques may also be seen. The most common anatomical location for these lesions in on the bilateral upper and lower extremities, abdomen, and chest
Indications for treatment of infantile hemangiomas with oral propranolol include large size, ulceration, and location (either in a cosmetically sensitive area or causing functional impairment). Oral propranolol is first-line and the treatment of choice for these particular scenarios (Leung et al. Curr Pediatr Rev. 2020). Propranolol is a nonselective beta-adrenergic receptor antagonist. In contrast, betaadrenergic agonists generally have the opposite function of beta blockers. Activation of β1 receptors induces positive inotropic and chronotropic output of cardiac muscle (increased heart rate and blood pressure), secretion of ghrelin from the stomach, and renin release from the kidneys. Activation of β2 receptors induces smooth muscle relaxation in the lungs, GI tract, blood vessels, and uterus. Understanding the mechanisms of beta-adrenergic receptor antagonists like propranolol vs. beta-adrenergic agonists is useful in remembering the side effects associated with oral propranolol. Several mechanisms of action have been proposed to explain propranolol's efficacy in the treatment of infantile hemangiomas. These potential mechanisms include vasoconstriction, endothelial cell apoptosis, and angiogenesis inhibition (Kum et al. Dermatoendocrinol. 2014). The standard dosing regiment is 1 mg/kg/day in divided doses with the goal of reaching 2-3 mg/kg/day (McGee et al. Ulster Med J. 2013). Common adverse effects of oral propranololtherapy include: Sleep disturbances Bradycardia Hypotension Hypoglycemia Additionally, reactive airway disease is a relative contraindication to oral propranolol therapy and should be screened for prior to initiation, as bronchospasm can be worsened. Heart rate and blood pressure should be monitored closely after the initial dose and after any dose increase. The risks of hypoglycemia can be minimized by administering oral propranolol during or immediately after feeding. Should the patient have an acute illness marked by vomiting or decreased oral intake, the dose of propranolol should be decreased or it should be temporarily discontinued
The pictured lesion is a nevus comedonicus. Androgen stimulation at puberty tends to exacerbate these lesions. Nevus sebaceus, Apert syndrome, and female androgenetic alopecia are also all affected by androgen stimulation. Eyebrow, eyelash, and vellus hair growth are relatively androgen-independent. (Yildiz et al. Hum Reprod Update 2010). Lateral and occipital scalp hair are also relatively androgen-independent. Nevus comedonicus pathogenesis is related to fibroblast growth factor-receptor gene 2 (FGFR2) mutations. FGFR2b-signaling is involved in dermal-epithelial interaction for skin appendage formation, pilosebaceous follicle homeostasis, comedogenesis, sebaceous gland proliferation, and lipogenesis.
Epidermal nevi (EN) are benign congenital hamartomas of epidermal structures. The description, history, and histology of the lesion is typical of inflammatory linear verrucous epidermal nevi or ILVEN. ILVEN are a specific type of EN, characterized by pruritus and unilateral linearly distributed verrucous papules coalescing into plaques, often along lines of Blaschko (D.). While there is a possible association between ILVEN and arthritis (rare reports), it has not been associated with the extra-cutaneous abnormalities seen in EN syndromes (see below) (A.). Although multiple therapies have proven effective, treatment can be difficult. Treatments which have been reported effective include but are not limited to topical or intralesional steroids, topical retinoids, 5-fluorouracil, topical calcineurin inhibitors, CO2 laser, Excimer laser, biologics (etanercept, adalimumab, ixekizumab), crisaborole, and surgical excision. While surgical excision is one of several treatment options, scarring is a limiting factor, therefore other reported treatment modalities should first be employed (C.). Subtypes of EN Nevus uniuslateris - extensive unilateral EN that may be single linear or spiral lesion, or multiple lesions forming an intricate pattern affecting multiple sites Systematized EN or ichthyosis hystrix - extensive bilateral lesions, primarily on the trunk ILVEN - blaschkoid lesions with chronic pruritus and are not associated with EN syndrome EN syndrome - group of complex disorders characterized by EN with associated extracutaneous abnormalities most often neurologic, ocular, and musculoskeletal. Examples include CHILD syndrome, Becker nevus syndrome, nevus comedonicus syndrome, phacomatosis pigmentokeratotica, Proteus syndrome, Schimmelpenning syndrome, type 2 segmental Cowden disease, Garcia-Hafner-Happle syndrome, and angora hair nevus syndrome
A 12-year-old girl is brought to your dermatology clinic by her mother. She complains of small bumps on her hands and arms. She has no significant past medical history. Her younger brother has similar lesions, but to a lesser extent. On exam, there are numerous flat-topped skincolored papules on the bilateral hands and arms. On her chest, there are multiple scattered scaly hypopigmented macules. Your clinical suspicion is that the patient may have an autosomal recessive disorder. What viruses are most commonly associated with this condition?
- mutation, pathophysiology, presentation, risk of transformation to what? treatment
Epidermodysplasia verruciformis (EV) is an autosomal recessive (AR) disorder caused by inactivating mutations in EVER1 (TMC6) and EVER2 (TMC8) gene, which encode transmembrane proteins in the endoplasmic reticulum (Huang et al. Int J Dermatol 2018). Many consider EV to be a form of intrinsic immunodeficiency with susceptibility to EV specific Many consider EV to be a form of intrinsic immunodeficiency with susceptibility to EV-specific HPVs, specifically the beta-papillomaviruses. Patients present with widespread tinea versicolor-like hypopigmented macules on the trunk or verruca plana-like flat papules, commonly affecting the hands, extremities, and face. Transformation into squamous cell carcinoma typically develops in adulthood and on sun-exposed sites, and is most commonly associated with HPV 5 and 8. An acquired form of EV is seen in HIV. Treatments for patients with EV include photoprotection, close surveillance, and anti-viral treatments such as imiquimod or cimetidine. HPV 1 and 2 are associated with common warts. HPV 3 and 10 are associated with verruca plana, or flat warts. HPV 7 and 2 are associated with Butcher's warts, which is most commonly seen in meat and fish-handlers HPV 16 and 18 are considered high-risk for malignant transformation and are associated with several malignancies (e.g., cervical, vulvar, anal, and head and neck cancers)
When seeing pediatric patients with acne, it can be useful to divide based on the age of the patient. Neonatal acne presents in the first few weeks of life and spontaneously regresses over 3 to 6 months of age. Infantile acne presents between 1 and 12 months of age. While both neonatal acne and infantile acne can present similarly, infantile acne tends to have open and closed comedones whereas neonatal acne typically does not. In some instances, infantile acne can even be nodulocystic and has a higher risk of scarring and if severe may require aggressive therapy with oral erythromycin or even isotretinoin. Mid-childhood acne (MCA) is defined as the onset of acne between 1 and 7 years of age. MCA is typically not present in isolation and is highly associated with an underlying endocrine disorder. Underlying disorders triggering MCA include late-onset congenital adrenal hyperplasia, androgen secreting tumors (from an ovarian or adrenal source), precious puberty, and exogenous androgen exposure. Hormonal evaluation or direct referral to endocrinology is warranted, with the recommended laboratory evaluation including serum testosterone (free and total), luteinizing hormone (LH), follicle stimulating hormone (FSH), dehydroepiandrosterone sulfate (DHEAS), 17-hydroxyprogesterone (17-OHP), prolactin, and radiographic bone age.
Neonatal lupus - affiliated with what maternal antibodies? - clinical presentation - extracutaneous involvement of what organ system?Neonatal lupus erythematosus (Lee et al. J Invest Derm 1993): Associated with maternal antibodies in the Ro/La family (mainly Ro/SSA) and U1RNP that cross trans-placentally. Anti-Ro/SSA target the cardiac conduction system. Anti-La/SSB antibodies may be more important for the development of skin disease. Anti-U1RNP antibodies are associated with atypical skin lesions and absence of cardiac disease About 1-2% of infants with a mother with these antibodies will develop cardiac disease Maternal MHC types HLA-B8 and DR3 confer an increased risk Cutaneous eruption is classic for the periorbital distribution with annular plaques and telangiectasias, but commonly also affects the cheeks and scalp as well. The skin findings typically resolve within six months without sequelae or scarring Heart block (in up to 30% of cases is the feared complication (usually starts as 2nd degree block and progresses to 3rd degree), and typically is noted at birth. Unfortunately, many of the heart blocks are permanent, and may require a pacemaker 10% of cases will have concurrent hepatobiliary disease Hematologic complications can also be seen: thrombocytopenia neutropenia and anemia Treatment is aimed at early detection (i.e. EKG) and intervention/treatment. Corticosteroids are often utilized Mothers are typically asymptomatic from underlying disease at birth Histopathology findings are comparable to those of LE in an adult and include interface dermatitis, necrotic keratinocytes, follicular plugging, and granular IgG deposition at the DEJ on DIFRubinstein taybi - gene, key findingsRubinstein-Taybi is a rare syndrome that is usually sporadic and related to mutations in the CREBBP gene (encodes a nuclear protein that activates cAMP regulated gene expression). In ~10% of patients this is due to a submicroscopic deletion of the CREBBP gene on chromosome 16p13. This disorder has classic findings which include: Profound growth restriction Severe intellectual disability Microcephaly. Patients also have characteristic facies with the following features: Broad nasal bridge Beaked nose (nasal septum below ala) Downslanting palpebral fissures Epicanthic folds Grimacing smile Micrognathia Other findings include short stature, broad thumbs and halluces, eye abnormalities (cataracts/strabismus) cryptorchidism, congenital heart defects, and respiratory infections. (Hsiung et al. Dermatol Online J 2004) Pertient dermatologic findings include the following: Capillary malformations(~50% of patients) Pilomatricomas Spontaneous keloid scarringCornelia de lange syndrome - gene - dermatologic findingsMost cases of Cornelia de Lange syndrome are sporadic with a mutation in NIPBL gene being present in some, but not all cases. Dermatologic findings include cutis marmorata, hirsutism,synophrys(eyebrows coalesce over the nasal root forming one brow), trichomegaly (long lashes), and low hairline. Patients have a distinct and characteristic facies with microcephaly, small nose with anteverted nostrils, downturned thin lips, long philtrum, micrognathia, low set ears, and a short neck.Familial Dysautonomia - gene, pathophysiology - cutaneous findingsFamilial Dysautonomia (aka Riley-Day syndrome) is an autosomal recessive (AR) disorder caused by an IKBKAP gene mutation causing arrested development of unmyelinated sensory and sympathetic neurons with autonomic dysfunction. Dermatologic findings include erythematousskin patches as a result of autonomic or gustatory stimulation, absent fungiform papillae, hyperhidrosis, and burn injuries secondary to an indifference to pain. Patients have the appearance of being alert due to a decreased blink reflex. The decreased blink reflex is due to decreased corneal sensation to tear flowRussell silver syndrome - dermatologic findingsRussell-Silver syndrome is a very rare syndrome that is usually sporadic and likely a heterogeneous group of disorders. Dermatologic findings include café au lait macules. Patients have a characteristic triangular facie swith down-turned thin lips(similar to Cornelia de Lange), micrognathia, and delayed anterior fontanelle closure.Treatment of tinea capitis? - what is not recommended - what are the best two first line agents?Treatment of tinea capitis should consist of an oral antifungal. Topical antifungals have inadequate hair follicle penetration and may not cover areas of subclinical disease, and are therefore much less likely to be effective (but may be used in combination with orals). Systemic antifungals that are effective for tinea capitis include: Griseofulvin Terbinafine Itraconazole Fluconazole Oral ketoconazole is NOT recommended, due to risk of liver toxicity. Of the effective agents, griseofulvin is considered by many to be first-line therapy, however,terbinafine has evolved to be an alternate first-line agent to griseofulvin.Birt Hogg Dube syndrome - gene, inheritance - cutaneous findings - noncutaneous findings - criteria for diagnosis?BHD is a rare genodermatosis inherited in an autosomal dominant(AD) pattern; as such, there is typically a family history of similar symptoms. Itis caused by a mutation in the FLCN gene (also referred to as BHD gene) on chromosome 17p11.2, which encodes the folliculin Clinical features of BHD include: Multiple fibrofolliculomas Acrochordons Trichodiscomas Patients are at risk of developing renal cancer (specifically early-onset <50 years old), and pulmonary cysts which can result in spontaneous pneumothorax European guidelines for diagnosis of BHD include fulfilling one major, or two minor criteria: Major criteria: 5 or more fibrofolliculomas or trichodiscomas (at least one confirmed by biopsy) of adult onset FLCN gene mutation Minor criteria: Multiple pulmonary cysts (often basilar) with no other apparent cause, with or without spontaneous pneumothorax Renal cell carcinoma with early onset < 50 years old First degree relative with diagnosis of BHD A helpfultip for cutaneous manifestations of BHD to remember is a "FAT hog": F - fibrofolliculomas A - acrochordons T - trichodiscomasGorlins syndrome - gene mutation, inheritence - presentation, predisposed to developing what forms of cancer LEOPARD Syndrome - gene and findingsPTCH1 is mutated in Gorlin's syndrome, also referred to as Basal Cell Nevus Syndrome. It is usually inherited in AD fashion. PTCH1 normally inhibits smoothened - the mutation results in uninhibited smoothened and activation of Gli. Patients typically present with multiple BCCs, odontogenic keratocysts of the jaw, bifid ribs, palmoplantar pits, calcification of falx cerebri, and are predisposed to developing medulloblastoma. PTPN11 is mutated in LEOPARD syndrome, and is usually inherited in AD fashion. Classic clinical features include lentigenes, EKG abnormalities, ocular hypertelorism, pulmonic stenosis, abnormal genitalia, growth retardation, and deafness.A 5-year-old male is brought to the pediatric dermatology clinic by his mother for evaluation of hair loss. Mother states that patient was born with normal-appearing hair. However, within a few months of life, his hair began appearing short, fragile, and brittle. Exam is notable for normal eyebrows, normal eyelashes, spoon-shaped nails, and hyperkeratotic erythematous papules along the bilateral upper arms. Exam of the scalp reveals diffuse short brittle hair with notable perifollicular erythema and hyperkeratosis. Trichogram of plucked hairs reveals elliptical nodules uniformly distributed along the shaft. - What is the most likely diagnosis and most common associated gene mutation? - most common nail association, most common skin findingsMonilethrix is derived from the Latin and Greek word for "necklace hair." Areas of alopecia characteristically exhibit perifollicular erythema and hyperkeratosis, and light microscopy or trichogram of hair samples will show uniform elliptical nodules of normal thickness and intermittent abnormal constrictions. Eyebrows, eyelashes, and nails may be affected. Among nail findings, koilonychia ("spoon nails") is the most common (Bolognia et al. Dermatology 2018). Keratosis pilaris is also commonly associated with monilethrix. Patients with monilethrix are born with normal-appearing hair that is replaced by brittle, short hair within the first few months of life. Monilethrix is typically inherited in an autosomal dominant (AD) manner with variable expression, though a recessive form has been described with mutations in DSG4 (Farooq et al. Br J Dermatol 2011). The AD inherited form involves mutations in hair cortex-specific keratin genes KRT86 (most often), KRT81 and KRT83.Epidermolytic hyperkeratosis - is the primary histologic feature of what condition - cause of condition? where are these mutated structures typically expressed?Epidermolytic hyperkeratosis (EHK) is the primary histologic feature of Bullous Congenital Ichthyosiform Erythroderma (BCIE). EHK is a distinct type of keratinocyte degeneration where the superficial epidermis is degenerated. The superficial epidermis looks like it was blown away by a shotgun. At low power, epidermolytic hyperkeratosis (EHK) is difficult to distinguish from the koilocytic change (the white halo around nuclei) and hypergranularity of viral warts. The affected cells in verruca are vacuolated but do not show epidermolysis EHK is also known as Bullous Congenital Ichthyosiform Erythroderma (BCIE). BCIE is an autosomal dominant (AD) genodermatoses caused by KRT1 and KRT10 mutations. KRT1 and KRT10 are expressed in the spinous and granular epidermal layers of the epidermis.Progeria - gene, pathophysiology - characterized by what? - characteristic facies? most common cause of death?Hutchinson-Gilford progeria syndrome is a genetic disorder associated with mutation in the LMNA gene encoding nuclear lamins A and C, which are two protein products that are major components of the nuclear membrane lamina which contribute to the structure of the nucleus. The mutation results in an abnormally farnesylated lamin protein, thereby affecting its ability to insert normally into the nuclear envelope. This leads to functional defects like abnormal DNA repair, mitochondrial impairment, aberrant cell-cycle regulation, shortened telomeres, altered epigenetic regulation and gene expression, and increased cellular senescence which result in premature aging. Patients with Hutchinson-Gilford progeria syndrome experience rapid onset of aging characteristics in early childhood (typically in the 1st decade of life around 6-18 months of age). Characteristic features including frontal bossing, enlarged head, high-pitched voice, alopecia, beaked or "bird-like" nose, micrognathia, prominent skin veins, and sclerodermatous skin tightening. Histopathology shows nonspecific thickened dermal collagen. The most common cause of death in these patients is complications of cardiovascular disease due to atherosclerosis, as well as cerebrovascular disease. Treatment primarily involves early interventions, screenings, and treatments surrounding avoidance of complications, particularly the more common CV and musculoskeletal complications. Other therapies which are being explored include farnesylation inhibitors, bisphosphonates, statins, aminopyrimidines, antisense oligonucleotides, rapamycin, isoprenylcysteine carboxyl methyltransferase inhibition, resveratrol, vitamin D supplementation, and remodulin. Other disorders in the differential diagnosis of premature aging include Werner syndrome (see below), acrogeria, menagerie, and Néstor-Guillermo progeria syndrome. A few other genodermatoses that may share some overlapping features include Cockayne syndrome (see below), Kindler syndrome, Rothmund-Thompson syndrome, Ataxia-telangiectasia, Cutis laxa with progerioid features, Ehlers-Danlos, and Wiedemann-Rautenstrauch syndrome.Werner syndrome - gene, inheritance - more common in what country? - when do symptoms onset? how does this compare to progeria - typical cause of deathRECQL2 gene mutation is associated with the premature aging syndrome, Werner syndrome. Werner syndrome is an autosomal recessive (AR) disorder with a defect in DNA helicase and is more common in Japan. Symptoms generally start in the 2nd decade of life (in contrast to Hutchinson-Gilford progeria syndrome which starts very early in life). Patients also have characteristic alopecia, beaked nose, micrognathia, and sclerodermoid changes. There is an increased incidence of premature cataracts, diabetes, hyperlipidemia, osteoporosis, atrophy, atherosclerosis, and increased risk of tumors and malignancy (meningioma, hepatoma, breast and ovarian carcinoma, thyroid adenocarcinoma, fibrosarcoma, osteogenic sarcoma, and skin cancer). The most common causes of death typically occurring in the mid-50s are malignancy and cerebrovascular/cardiovascular events. BLM gene defect is associated with Bloom syndrome (congenital telangiectatic erythema), an AR condition resulting in disruption of ATP-dependent DNA helicase activity, and subsequent sister chromatid exchange and chromosomal breaks and rearrangements. Clinical features include photosensitivity, poikiloderma (butterfly distribution on nose and cheeks and eruptions on the ears, forearms, and dorsal hands), facial dysmorphism (long narrow face with a prominent nose, small mandible and malar hypoplasia), hypogonadism, high-pitched voice, and increased risk of infections (decreased immunoglobulin levels), and high-risk of malignancies (leukemia, lymphoma, GI adenocarcinoma, oral/esophageal SCC). Bloom syndrome, Werner syndrome, Xeroderma pigmentosum (XP), and Rothmund-Thomson syndrome share helicase family mutations.Most common cause of tinea capitis in the US, UK, candada, brazil, france, and japan? - other causesTrichophyton tonsuransis the most common cause oftinea capitis in the United States. It is also the most common cause in the the United Kingdom, Canada, Brazil, France, and Japan. Tinea capitis is generally caused by organisms in the Trichophyton and Microsporum genera, and rarely caused by Epidermophyton species. The most common dermatophyte causing tinea capitis varies by location: United States and the United Kingdom: T. tonsurans Canada, Brazil, France, and Japan: T. tonsurans Central and southern Europe and Russia: M. canis Australia, and China: M. canis Turkey: T. verrucosum West Africa: M. audouinii and T.soudanenseEhlers-Danlos syndrome - typical cutaneous manifestations - what is gorlins sign? present in what % of patients - absence of what can be seen - name the types of ehlers danlos?Ehlers-Danlos syndrome is a heterogeneous group of disorders caused by various collagen, collagen enzyme, or collagen associated protein defects. Typical cutaneous features include the following: Hyperextensible and fragile skin Easy bruising Cigarette-paper-like scarring Gaping/fish mouth scars Hypermobile joints Hernias Blue sclerae Gorlin's sign (ability to touch tongue to tip of nose. Seen in 50% of Ehlers-Danlos and 10% of normal population) Absence of lingual frenulum Classical EDS, vascular EDS, athrochalasia EDS, dermatosparaxis EDS, and cardiac-valvular EDS are disorders of collagen primary structure and collagen processing, whereas kyphoscoliosis EDS is a disorder of collagen folding and cross-linking (see below for a more in depth understanding of the function of lysyl hydroxylase). Three alpha chains form a collagen triple helix. Each alpha chain is enzymatically hydroxylated by lysyl hydroxylase and prolyl hydroxylase (requires vitamin C, O , Fe , and aketoglutarate) and glycosylated intracellularly. The three alpha chains then attach, twist, and propagate from the C terminal region to the N terminal region forming procollagen. (Myllyharju et al. Trends Genet 2004) Procollagen is then transported extracellularly and the N and C terminal propeptides are cleaved. It is extracellularly that collagen cross-linking and assembly into larger collagen structures occurs, a process which is catalyzed by enzymes including lysyl oxidase (copper dependent) and transglutaminase. The different types of collagen form different suprastructures, shapes and networks to fulfill their specific functions.Tuberous sclerosis - gene? what do these genes do? - associated with what hamartomatous lesions?Tuberous sclerosis complex is an autosomal dominant disorder that occurs secondary to a mutation in the TSC1 or TSC2 gene (both tumor suppressor genes). TSC1 encodes for the protein hamartin, whereas TSC2 encodes for tuberin. Hamartomas of multiple organs (including the brain, heart, retina, and liver) Subependymal giant cell tumor Cardiac rhabdomyoma Seizures Autism and behavioral disorders Pulmonary disease (e.g. interstitial fibrosis and pulmonary cysts) Ophthalmic disease (retinal and non-retinal disorders) Renal tumors (angiomyolipomas and renal cell carcinomas) Clinical Pearl: Tuberous sclerosis complex results from mutation in the TSC1 or TSC2 gene and patients can present with a variety of signs and symptoms. Patients may develop a variety of tumors including hamartomas of several organs, subependymal giant cell tumors of brain, cardiac rhabdomyomas and renal tumors (angiomyolipomas and renal cell carcinoma)- Most common mutation in mastocytosis? - this gene is also mutated in what condition? how is the mutation different than in mastocytosis?Activating mutations have been found in 86% of children with mastocytosis. In this disease entity, pluripotent (CD34+) bone marrow cells circulate as monocytic agranular cells, which then mature into fully granulated cells in tissues including the liver, spleen, lymph nodes, GI tract, and skin. The most common genetic mutation in mastocytosis is an activating mutation in c-KIT. KIT is the protein product of c-KIT and this ligand-independent activation of KIT induces cellular growth and extends mast cell survival by preventing apoptosis. Piebaldism usually results from a mutation in the c-KIT proto-oncogene, which encodes a member of the tyrosine-kinase family of transmembrane receptors found on melanocytes. Inactivating mutations in c-KIT leads to abnormal development of melanocytes before and after melanoblast migrationNeonatal acne - cause of disease? resolution usually occurs when? this correlates with what? - what is the treatment for neonatal acne - what is the work up for mid childhood acne?Neonatal acne is due to transient sebaceous gland stimulation and typically self-resolves and does not require endocrinology evaluation. Neonatal acne may be caused by stimulation of sebaceous glands by maternal androgens or a transient androgen production. In a subset of these patients (neonatal cephalic pustulosis) Malassezia spp may be found on KOH preparations. Sebaceous gland production decreases around 3-6 months, which coincides with the resolution of neonatal acne. Treatment, if indicated, includes gentle cleansing with soap and water, topical ketoconazole, topical retinoids, topical antibiotics, or systemic antibiotics/isotretinoin if severe. If acne arises between 1-7 years of age, care should be taken to evaluate for other signs of pubertal development. Signs of thelarche, pubarche, etc. should be assessed and an endocrinology workup is indicated. Laboratory evaluation would include DHEAS, androstenedione, 17-OH-progesterone, and bone age. Comedones and inflammatory cysts are a finding of infantile acne, which presents at a later age than neonatal acne (around 3-6 months) and is due to a hormonal imbalance. Treatment is indicated in these cases to prevent scarring. Clinical findings in neonatal acne include monomorphic erythematous papules and pustules, but not comedones or inflammatory cysts. These lesions generally heal without scarring and don't mandate treatment unless the presentation is severe.Hailey Hailey disease - aka, gene, this gene is responsible for what - typically presents when? presentation - classic histo finding? - First line treatmentThis is a classic case of Hailey-Hailey disease (HHD) (benign familial pemphigus, benign chronic pemphigus); an autosomal dominant (AD) disorder which alters adhesion of epidermal keratinocytes. This disorder is caused by a loss-of-function mutation in the ATP2C1 gene at 3q22.1 leading to altered function of an ATP-powered pump that typically sequesters calcium into the golgi appartus thus leading to dysregulation of cytoplasmic calcium in keratinocytes. (Hu et al. Nat Genet 2000) The classic presentation is a 20-30 year old patient with new onset macerated plaques, and/or superficial vesicles and erosions within the intertriginous areas. Secondary infections are frequently observed in this condition. This disease typically waxes and wanes in its severity. (Gisondi et al. Acta Derm Venerol 2005) On histopathologic review there is a loss of adherence among suprabasal keratinocytes (acantholysis), leading to vesicles and bullae. The classic finding in this condition occurs when there is partial acantholysis leading to the formation of layers of intraepidermal and suprabasal, detached keratinocytes which resembles a dilapidated brick wall. In addition, as opposed to autoimmune pemphigus, IgG is not present on DIF. If there is ambiguity following histopathological review, definitive molecular testing is available. The primary histologic finding in the associated histology image is acantholysis. Acantholysis refers to the loss of keratinocyte cohesiveness. This is secondary to dissolution of intercellular keratinocyte desmosomal connections. Acantholysis is a primary histologic finding in: Darier's disease Grover's disease (transient acantholytic dermatosis) Hailey-Hailey disease Pemphigus Warty dyskeratoma Management of this condition is typically supportive with an emphasis on controlling exacerbating factors. First-line treatment typically consists of topical corticosteroids and antibiotics.Name the major features of tuberous sclerosisMajor Features: Facial angiofibromas ≥ 3 or fibrous cephalic plaque Non-traumatic periungual or ungual fibromas ≥ 2 (Koenen tumors) Shagreen patch (a connective tissue nevus; typically in lumbosacral area) Three or more hypomelanotic macules ≥ 5mm (small polygonal or thumbprint shaped; ash leaf spots) Cardiac rhabdomyoma(s) (tend to involute by age 3) Cortical dysplasia (tubers and/or white matter radial migration lines) Lymphangioleiomyomatosis (typically asymptomatic) Multiple retinal nodular hamartomas Renal angiomyolipomas ≥ 2 (most are asymptomatic) Subependymal giant cell astrocytoma Subependymal nodules Minor Features: Confetti spots (guttate hypopigmented skin lesions; rare; <5% affected infants; most specific pigmentary finding) Dental enamel pits that are multiple ≥ 3 and randomly distributed Gingival fibromas ≥ 2 Multiple renal cysts Non-renal hamartomas Retinal achromic patch The diagnostic classifications are as follows: 1. Definitive diagnosis of TS: a patient has two major features or one major feature and two minor 2. Probable diagnosis of TS: a patient has one major feature and one minor feature 3. Possible diagnosis of TS: one major feature or two minor features Based on the patient's multiple facial angiofibromas (major feature) ≥ 2 gingival fibromas and ≥ 3 randomly distributed dental enamel pits (minor features) this patient has a definitive diagnosis of TS.A 48-year-old male patient has the pictured findings involving the bilateral axillae on physical exam. In addition to these findings, what additional cutaneous sign(s) would allow this patient to meet diagnostic criteria for neurofibromatosis type I? A. Six or more café au lait macules measuring greater than 5 mm in diameter B. Six or more café au lait macules measuring greater than 15 mm in diameter C. One neurofibroma D. None of the above, Neurofibromatosis 1 cannot be diagnosed on skin findings alone Name the gene, cutaneous findings - NF1 + JXG = ?The correct answer is B) Six or more café au lait macules measuring greater than 15 mm in diameter. Neurofibromatosis 1 (NF1) is an autosomal dominant (AD) disorder with an estimated incidence of 1 in 3000 births. Interestingly, genetic penetrance of the disease is nearly 100%, but expressivity varies resulting in different findings and complications in individuals with the same disease. The implicated NF1 gene encodes the neurofibromin protein, which is a negative regulator of the Ras-MAPK (mitogen-activated protein kinase) pathway. As such, the NF1 gene is considered a tumor suppressor gene. Clinical features of NF1 can span multiple organ systems, with potential for cutaneous, ocular, skeletal, neurologic, and cardiovascular involvement. Cutaneous findings consist of neurofibromas, café au lait macules (CALMs), plexiform neurofibromas, nevus anemicus, and juvenile xanthogranulomas. There is an increased risk of juvenile myelomonocytic leukemia in young pediatric patients with NF1, and there may also be a further increased risk in pediatric patients with co-existence of NF1 and juvenile xanthogranulomas.Of the following photosensitive disorders, which is the result of a defective nucleotide excision repair gene? A. Cockayne Syndrome ; B. Rothmund-Thomson Syndrome C. Bloom Syndrome D. Phenylketonuria E. Hartnup Disease Name the disease, name the gene, clinical manifestationsCockayne syndrome is an autosomal recessive (AR) disorder cause by the defective nucleotide excision repair genes ERCC6 (most common) or ERCC8. Clinical features include photosensitivity without pigmentary change which helps differentiate from Rothmund-Thomson and Bloom syndromes. Characteristically patients are of short stature with loss of adipose tissue and thin body habitus. Ears are often large or prominent and deafness or mental retardation may be present Retinopathy basal ganglia calcification and dental caries commonly occur. Patients often have reduced life span with expectancy into the 40's due to neurodegenerationWhat is the best imaging test to screen for liver hemangiomas in a patient with multiple cutaneous infantile hemangiomatosis? Most common mutation in patients with tuberous sclerosis? What condition has been shown to be associated with a lack of a frenulumabdominal ultrasound TSC2 The image demonstrates the absence of the lingular frenulum. Absence of the lingular frenulum has been shown to have a 71.4% sensitivity and 100% specificity for classical and hypermobility types of Ehlers-Danlos syndromeA 12‐month‐old male was referred to your clinic by his pediatrician due to bilateral, symmetrical lesions on his soles. The child has no past medical history and is healthy. On exam, there are skin-colored nodules measuring ~1 cm on the medial aspect of his bilateral heels. The lesions are nontender and nonpruritic. The lesions do not accentuate with standing. His parents believe the lesions presented "when he was a few months old" and have "gotten a little bigger as he has gotten older". Family history is negative. What is the next step in management? A. Biopsy B. Ultraound C. Observation D. Referral to oncology? Name the condition, most common age of presentation, classic location, do they grow?c. Observation This vignette demonstrates a case of infantile pedal papules. This is a benign disorder and you should recommend observation, as the lesions will resolve spontaneously over time. Infantile pedal papules have been referred to by various names in published papers: Bilateral congenital adipose plantar nodules Precalcaneal congenital fibrolipomatous hamartomas Benign anteromedial plantar nodules of childhood Congenital piezogenic-like pedal papules Bilateral congenital fatty heel pads Pedal papules in the newborn Infantile pedal papules are common, benign papules that can present at birth but more commonly present in the first few months of life. The most common age of presentation is 1- year-old and the lesions typically resolve by age 2 to 3 years old. The etiology and pathogenesis of this disorder is unclear. The papules are generally skin-colored and soft. Additionally, they are generally asymptomatic: nontender, nonpruritic, and do not affect ambulation. Infantile pedal papules generally present as solitary lesions that are symmetric (involving both feet) and measure 5 to 10 mm in size. They are classically located on the medial aspect of the heel and can extend anterior to the heel. Additionally, as demonstrated in this vignette, the papules can increase in size as the child grows. They are not associated with any systemic disorders and management is observation, as they will resolve with time. A biopsy is not recommended/indicated. However, if a biopsy is performed, histology will reveal mature and diffuse adipose tissue in the hypodermis and reticular dermis surrounded by collagenous, nonhypertrophic, fibrous septa. No inflammatory changes will be present, however mucinous deposits are commonly seen within fat lobules at the periphery. Nerve fiber size and number is normal, as is the distribution and number of eccrine glands.Cornelia de lange syndrome - inheritance and gene - characterized by what featuresCornelia de Lange syndrome follows an autosomal dominant (AD) mode of inheritance most commonly caused by mutations in NIPBL gene, RAD21 gene, or SMC3 gene. In addition, this syndrome can also be caused by mutations in the HDAC8 or SMC1A gene, and when this occurs its mode of inheritance is in a X-linked dominant pattern. Cornelia de Lange syndrome is characterized by the following: Short stature Intellectual disability Abnormalities of the bones in the arms, hands, and fingers (upper extremities). In addition, patients have a distinct and characteristic facies with the following features: Microcephaly Small nose with anteverted nostrils Downturned thin lips Long philtrum Micrognathia Low set ears short neck. Pertinent dermatological findings include the following: Cutis marmorata Hirsutism Synophrys(eyebrows coalesce over the nasal root forming one brow) (see images) Trichomegaly (long lashes) (see images) Low hairlineWhat are the dermatologic findings of arsenic poisoning? What is eruptive hypomelanosis? - seen in whom, possible etiology, course of disease what are Bier SpotsArsenicosis can cause sharply marginated porcelain-white macules similar to IGH. It is important to recognize other signs of chronic arsenic exposure, as long-term complications may occur (cognitive impairment, lung disease, cardiovascular disease/MI, liver damage, cancer, pregnancy complications). Dermatologic findings in arsenic poisoning include PPK, facial edema, transient flushing, morbilliform eruptions, acral desquamation, ungual changes (Mee's lines and Beau's lines), periungual pigmentation, and mucosal hyperpigmentation. Eruptive hypomelanosis is a disease of childhood characterized by the eruption of symmetric uniform hypopigmented macules with powdery scale. Lesions follow coryzal symptoms and may be associated with lymphadenitis and pharyngitis, suggesting an infectious etiology. Lesions are most commonly observed on the extensor extremities. Eruptive hypomelanosis spontaneously resolves and no treatment is required. (Zawar et al. JAMA Dermatol 2014) Bier spots are physiologic transient asymptomatic irregular anemic macules that typically present in young adults. Lesions are most commonly observed on the extensor extremities. They are the result of physiologic vascular abnormalities. Bier spots are usually idiopathic, but may be associated with other conditions (vascular abnormalities, hypercoagulable states, pregnancy, renal crisis in scleroderma, and palmar hyperhidrosis)A prominent red reflex is seen in what condition What is the earliest manifestation of gorlins syndrome Which autoinflammatory condition does not carry an increased risk of secondary amyloidosistyrosinase negative occulocutaneous albanism odontogenic cysts Hyper IgD syndromeIris nevi - what are they? - can look similar to what? this findings is found in what two conditionsThis patient has the classic findings of an iris nevus. Iris nevi are focal areas of flat pigmentation with minimal disruption of the normal iris. They can be circumscribed as seen in this case, or diffuse. Of note, sectoral heterochromia iridis may present similarly to an iris nevus. In complete Heterochromia, one iris is a different color from the other. Sectoral heterochromia iridis refers to part of one iris being a different color from the remainder of the iris. Heterochromia iridis can be inherited, caused by genetic mosaicism, chimerism, disease, or injury. Heterochromia iridis can be seen in genodermatoses not limited to Nail-Patella syndrome and Waardenburg syndromePseudoxanthoma Elasticum -gene, pathogenesis - eye findings of PXE - cutaneous findings, path findings - systemic complicationsThe patient in the vignette above has pseudoxanthoma elasticum (PXE). PXE occurs secondary to decreased activity of anti-mineralization proteins, leading to calcified and fragmented elastic fibers, and their degradation. PXE is caused by mutations in ABCC6 which is a transmembrane transporter which may be involved in transporting a g-glutamyl carboxylase cofactor from the liver into circulation. (gglutamyl carboxylase catalyzes the activation of matrix gla protein (MGP). MGP is an inhibitor of pathologic mineralization. Eye findings in pseudoxanthoma elasticum (PXE) include rupture in Bruch's membrane causing angioid streaks, macular degeneration, retinal pigment alterations, retinal hemorrhage, and blindness. Histologically, pseudoxanthoma elasticum demonstrates clumped, calcified elastic fibers in the dermis. This is a rare genetic disorder with grouped yellowish papules overlying lax or soft/redundant skin typically on the neck or flexural areas giving a "plucked chicken" appearance (see associated image). Yellow papules can also be observed on mucosal surfaces. Systemic complications include gastric artery hemorrhage, hematemesis, epistaxis, myocardial infarction (MI), Cerebrovascular accident (CVA), angina, hypertension, claudication, and mitral valve prolapseLipschuts ulcers - what are they - common presentation - associated with what virus? - course of lesions - EBV is what type of virus? it has been linked to what other conditions?Lipschutz ulcers (non-sexually related acute genital ulcers) present acutely as exquisitely tender genital ulcers without any evidence of sexually transmitted infection (STI). These lesions have been associated with Epstein-Barr Virus (EBV), though most cases do not have an identifiable etiologic cause. The common history given is the acute presentation of exquisitely tender vulvar/vaginal ulcers in non-sexually active or pre-pubertal girls. Some patients may have prodromal symptoms including fever, chills, and lethargy however the majority do not have any associated symptoms. The lesions last for approximately 1-2 weeks and resolve without necessary interventions. Despite the benign course of the lesions, Lipschutz ulcers are a diagnosis of exclusion and other causes of urogenital ulceration must be ruled out. (Di Lernia et al. Int J Dermatol 2013) (Farhi et al. Arch Dermatol 2009) EBV is a human herpes type 4 virus which is a double-stranded DNA virus. It is spread through person-to-person contact including saliva, blood, transfusion products, and transplantation. EBV has been associated with a large number of diseases. Pertaining to dermatology, some include: Oral hairy leukoplakia, mononucleosis, non-sexually related acute genital ulcers/Lipschutz ulcers, histiocytic necrotizing lymphadadenitis, hydroa vacciniforme, and less-commonly Gianotti-Crosti syndrome (this is also associated with hepatitis B virus).What condition is associated with tomato catsup on fundoscopic exam?sturge weber syndrome PMH is a common skin disorder that is frequently misdiagnosed. Fluorescence localized to the follicles is pathognomonic of PMH. PMH is characterized by poorly marginated hypopigmented macules that favor the trunk, but may involve the proximal extremities and head/neck. Cutibacterium acnes(formerly Proprionibacterium acnes) bacteria within hair follicles has been proposed as the cause of PMH, through the production of an unknown depigmenting factor. Treatments includes UVA, topical benzoyl peroxide and topical clindamycin. (Relyveld et al. Am J Clin Dermatol 2007) A Wood's light examination should always be performed in acquired disorders of hypopigmentation. A review of acquired disorders of hypopigmentation, algorithm for diagnosis, and treatment options can be found herePatients with pseudoxanthoma elasticum should have regular follow up with what two specialties? Patients with an epidermal nevus and pathology demonstrating epidermolysis and hyperkeratosis can have children at risk for developing what?ophthalmology and cardiology Bullous congenital ichthyosiform erythroderma - Epidermolytic Hyperkeratosis (EHK), also known as BCIE, is an autosomal dominant (AD) genodermatosis caused by KRT1 and KRT10 mutations, which are keratins expressed in the spinous and granular epidermal layers of the epidermis. BCIE presents at birth with widespread erythroderma and denuded skin. Over time, the skin becomes hyperkeratotic with deep ridges favoring the flexural surfaces and cobblestone appearing hyperkeratosis involving the extensor surfaces. Bacterial colonization of the skin can cause a distinctive odor. Histologic findings include dense hyperkeratotic orthokeratosis, acanthosis, hypergranulosis, and a distinct degeneration of suprabasal and granular layer keratinocytes which exhibit vacuolization and dense clumps of keratin. Small intraepidermal blisters may also be observed.Lamellar ichthyosis - caused by a deficiency in what? what does this enzyme usually do - can present with what at birth - typical presentationHer disease is currently very well controlled as can be observed in these clinical photos. Lamellar ichthyosis (LI) is caused by transglutaminase deficiency (Genes: TGM1, ABCA12, ALOX12B, ALOXE3, NIPAL4). Transglutaminase-1 catalyzes calcium dependent cross linking of structural proteins (involucrin, loricrin, desmosomal proteins, small proline-rich proteins, and keratins) and protein/hydroxyceramide bonds in the upper layers of the epidermis. The inability to form a normal cornified cell envelope (due to disturbance of both cornification and desquamation) leads to the clinical phenotype. Lamellar ichthyosis is present at birth (collodion membrane) and persists throughout life. During the neonatal period, LI is difficult to distinguish from other collodion membrane disorders including congenital ichthyosiform erythroderma (CIE) and the self-healing collodion baby (Characteristic changes that occur later in life make LI easily distinguishable from other ichthyoses). Lamellar ichthyosis demonstrates non-erythrodermic, large plate-like scales that look like the bark on a tree. Patients can also demonstrate the following findings: Eclabium (the turning outwards of a lip) Ectropion (the turning outwards of an eyelid) which can lead to keratitis, which may be treated with topical tazarotene (Hanson et al. Pediatr Dermatol 2017) Hypoplasia of nasal and articular cartilage Heat intolerance Recurrent ear infections Nail dystrophy Palmoplantar keratoderma (PPK)Schimmelpenning syndrome - aka - characterized by what? mutation - extracutaneous manifestationSchimmelpenning syndrome (SS), which is also called nevus sebaceus syndrome and Schimmelpenning-Feuerstein-Mims syndrome, is a rare, neurocutaneous disorder characterized by a linear nevus sebaceus, which often presents along the lines of Blaschko, with extracutaneous abnormalities that can be neurologic, skeletal, cardiovascular, ophthalmic, and/or urologic. It is caused by an autosomal dominant mutation in HRAS or KRAS. This mutation is lethal, so all cases are sporadic and patients are extremely unlikely to have a positive family history. Nevus sebaceus (a waxy reddish-orange, hairless, plaque that generally occurs on the scalp or face) is a hallmark of SS. Of note, approximately two-thirds of cases of nevus sebaceus present as a scalp lesion and approximately one-third as a facial lesion. Additionally, central nervous system and ocular involvement is common and can include seizures, intellectual disability, hemimegalencephaly, ocular choristomas and colobomas, and microphthalmos. Lastly, skeletal involvement can include short stature, craniofacial abnormalities, hypoplastic bones, and vitamin D-resistant hypophosphatemic ricketsChild syndrome - what does it stand for? - cutaneous findings? what is usually sparedCHILD (congenital hemidysplasia with ichthyosiform erythroderma and limb defects) syndrome is a rare disorder defined by birth defects of skin, viscera, musculoskeletal system, and CNS. Cutaneous findings include unilateral, waxy, scaling, erythematous ichthyosiform plaques with a sharp midline demarcation and ipsilateral underdeveloped limbs. The cutaneous pattern can be diffuse and/or linear, with streaks following the lines of Blaschko. However, the face is usually spared. Biopsy reveals psoriasiform epidermis with hyperkeratosis and parakeratosis and the papillary dermis usually has foam cells that express surface markers like CD68 and S100 proteinsCongenital syphilis - describe the titer criteria for diagnosis - early vs late disease findingsFinally, you must understand how to diagnosis congenital syphilis. While physical findings alone can yield a very probable diagnosis, in practice, it is desirable to have serologic evidence confirming one's diagnosis when considering a systemic treatment and it is recommended that all infants potentially exposed to syphilis in utero undergo serological testing. One of the diagnostic criteria for highly probable congenital syphilis is a neonatal titer that is four fold or more than that of the mother's corresponding titer, e.g. 1:32 vs 1:8. Other criteria include identifying the organism on histolopathologic examination (confirmatory) and classic clinical signs and symptoms (highly probable). Congenital syphilis occurs when the infectious spirochetes pass from the mother to the fetus. The disease can be divided in to early (first few months) and late (after 2 years). Findings suggestive of early congenital syphilis include: Fever Wasting Adenopathy and hepatosplenomegaly Skin eruptions (vesiculobullous, macular, popular and/or petechial) Rhinitis (snuffles) Hutchinson's triad (keratitis, notched central incisors, cranial nerve eight deafness) Findings of late congenital syphilis include: Saddle nose deformity Mulberry molars Eye abnormalities Joint swelling Anterior bowing of shins Unilateral thickening of the medial 1/3 of clavicleAlkaptonuria -eye findings - inheritance, pathophysiology - clinical findingsThis is the classic ocular presentation of alkaptonuria. Common ocular manifestations of alkaptonuria include symmetric bluish-black discoloration of the conjunctiva, cornea, and sclera. (Damarla et al. Indian J Ophthalmol 2017) (Linder et al. BMC Ophthalmol 2014) Alkaptonuria (ochronosis) is an autosomal recessive (AR) disorder due to deficient homogentisic acid oxidase, in which homogentisic acid cannot be metabolized (homogentisic acid is an intermediate in phenylalanine and tyrosine metabolism). Accumulation of homogentisic acid in cartilage, skin, eyes, body fluids, and many other connective tissues results in hyperpigmentation of eye/cartilage/skin etc., dark urine (when urine is left standing) with increased risk of renal calculi, and accumulation in joints leading to arthritis.Cystinosis - inheritance. most commonly seen in what nationality - most common form - findingsCystinosis is inherited in an AR pattern. It is most common in French-speaking Canadians. There are three main clinical forms of cystinosis depending on the age of presentation: infantile, adolescent, and adult. The infantile form is the most common. Corneal crystals appear as early as 1 year of age and are deposited in the anterior stroma. In patients with severe renal impairment (nephrotic form), patchy depigmentation of the retinal pigment epithelium can occur.Exanthem subitum - aka - cause of disease - course of disease and clinical findings - enanthemExanthem Subitum (Roseola Infantum; Sixth Disease) is a common febrile illness that typically affects infants and toddlers caused by HHV-6 and occasionally HHV-7. Infection with HHV-6 does not always lead to the classic findings in exanthem subitum. Exanthem subitum is characterized by high fevers with potential for seizures. Following resolution of the fevers, erythematous rose-pink colored macules and halo papules appear on the trunk, neck, and proximal extremities. Systemic symptoms can include URI, tympanic membrane infection, adenopathy, palpebral edema (Berliner's sign), and bulging anterior fontanelle. An enanthem of red macules and/or streaks involving the uvula and soft palate (Nagayama spots) and palatoglossal junction ulcers may develop. (Chua et al. J Clin Virol 2000). It typically resolves spontaneously after a few days with no sequelae.Sclerema neonatorum - characterized by what - typically affects whom? - prognosis - how is it differentiated from subcutaneous fat necrosis on histology?Sclerema neonatorum - Sclerema neonatorum is a rare, often-fatal skin disorder that is is marked by diffuse hardening of the subcutaneous tissue with minimal inflammation. It is symmetric and affects areas with higher amount of underlying adipose tissue and spares areas with less (e.g. palms, soles, and genitalia). It typically affects premature, newborn infants in their first week of life. Furthermore, these newborns present with comorbid illnesses including sepsis, dehydration, severe respiratory or gastrointestinal disease. Prognosis is poor, and it is estimated that only ~15 to 40% of patients survive. Like SCFN, histopathology demonstrates needle-shaped clefts within adipocytes. However, unlike SCFN, inflammatory cells (eosinophils, multinucleated giant cells, and histiocytes) are absent.Hailey Hailey Disease - aka, gene, pathophysiology - presents at what age? clinical findings - histology - First line therapyHailey-Hailey disease (HHD) (benign familial pemphigus) (benign chronic pemphigus) is an autosomal dominant disorder which alters adhesion of epidermal keratinocytes. This disorder is caused by a loss-of-function mutations in the ATP2C1 gene at 3q22.1 leading to altered function of an ATP-powered pump that typically sequesters calcium into the golgi appartus thus leading to dysregulation of cytoplasmic calcium in keratinocytes. (Hu et al. Nat Genet 2000) The classic presentation is a 20-30 year old patient with new onset macerated plaques of superficial vesicles and erosions within the intertriginous areas. Secondary infections are frequently observed in this condition. This disease typically waxes and wanes in its severity. (Gisondi et al. Acta Derm Venerol 2005) On histopathologic review there is a loss of adherence among suprabasal keratinocytes (acantholysis), leading to vesicles and bullae. The classic finding in this condition occurs when there is partial acantholysis leading to the formation of layers of intraepidermal and suprabasal, detached keratinocytes which resembles a dilapidated brick wall. In addition, as opposed to autoimmune pemphigus, IgG is not present by DIF. If there is ambiguity following histopathological review definitive molecular testing is available. Management of this condition is typically supportive with an emphasis on controlling exacerbating factors. First-line treatment typically consists of topical corticosteroids and antibioticshow is a solitary mastocytoma typically diagnosed? - clinical presentation - name some mast cell stains - what symptoms should be inquired about when someone presents with mastocytosis? - patients should be counseled to avoid what?Solitary cutaneous mastocytoma is a clinical diagnosis based on the typical lesion morphology, young age of presentation (birth or infancy), and positive Darier sign (urticates spontaneously in response to firmly rubbing or stroking the lesion). Lesions are indurated erythematous redbrown or yellow-brown papules, plaques, or nodules. They often have a peau d'orange rubbery or leathery consistency. When the diagnosis is in question, a skin biopsy can be obtained. Typical histopathology will demonstrate perivascular or diffuse dermal mast infiltrates and often eosinophils. Subepidermal bullae and dermal edema may also be observed. Mast cell stains including toluidine blue, Giemsa, Leder, and monoclonal antibodies that recognize tryptase or CD117 (KIT) can be helpful. Systemic symptoms are typically absent and lesions usually spontaneously resolve before puberty. When systemic signs/symptoms are present, other diagnoses should be considered. When a patient presents with cutaneous mastocytosis, you should inquire about abdominal pain, diarrhea, weight loss, bone pain, fractures, fevers, and malaise. Lesions are typically asymptomatic and will spontaneously resolve by puberty. Parents and patients should be counseled to avoid triggering factors including environmental triggers (friction, overheating) and mast cell degranulators (NSAIDs, aspirin, anticholinergics, narcotics, polymyxin B, alcohol, systemic anesthetics). Topical and intralesional corticosteroids may be used to control pruritus when lesions are symptomatic. Systemic agents can be used as well but are rarely required for solitary mastocytomas.You are called to the emergency department to evaluate a newborn that presented for swollen abdomen, failure to pass stool 48 hours after birth, and vomiting. On exam you note a white forelock and heterochromia irides. What is the most likely gene mutation in this newborn? - inheritance - name the 4 subtypes of disease and the genes associated with each - which type is associated with limb abnormalities? which is associated with hirschsprung disease?Waardenburg syndrome is an autosomal dominant(AD) disorder which affects the embryonic development of melanocytes, leading to areas of patchy depigmentation. (Read et al. J Med Genet 1997) It can be divided into 4 major sub-types characterized by various combinations of clinical features and genetic mutations. The four types are as follows: WS 1: Classic form characterized by dystopia canthorum. (PAX-3 mutation) WS 2: Heterogeneous group which typically lacks dystopia canthorum. (MITF gene mutation) WS 3 (Klein-Waardenburg syndrome): Limb abnormalities - hypoplasia, syndactyly. (PAX-3 mutation) WS 4 (Shah-Waardenburg syndrome): Hirschsprung disease. (SOX-10 , EDNRB, or EDN3 mutation) Common features of Waardenburg syndrome include the following: White forelock (most common pigmentary abnormality in Waardenburg) Heterochromia irides (multicolored iris) Synophrys (unibrow) Broad nasal root Dystopia canthorum (increase distance between inner canthi, with normal inter-pupillary distance).- calcium can be highlighted with what stain? - what are the 5 broad categories of calcium deposition - connective tissue disease cause which type of calcinosis? name the specific conditions -The dermatopathology photos shown demonstrate calcinosis cutis, or calcium deposition in the skin. These purple calcium deposits can be further highlighted with a Von Kossa stain. Historically, calcium deposition in the skin has been divided into 5 broad categories: Dystrophic Metastatic Idiopathic Iatrogenic Mixed (combination of metastatic and dystrophic) Dystrophic calcinosis cutis occurs after trauma to the skin which results in subsequent inflammation. In all forms of dystrophic calcinosis cutis (and contrary to metastatic calcinosis cutis and mixed calcification), the patient will have normal calcium and phosphorus metabolism. The question stem points to dystrophic calcinosis cutis secondary to prior heel-stick procedures. This is a common scenario seen in infants that have graduated from the NICU. Clinically, these lesions appear as firm chalky-white to yellow papules on the heel, due to the repeated heel-sticks. Notably, dystrophic calcinosis cutis can also be seen in juvenile dermatomyositis scleroderma (particularly the CREST variant), and systemic lupus erythematosus, all which can cause predisposition to calcium deposition in the skin, though fingers, elbows, knees, and shoulders are the most common locations for these conditions. In this clinical scenario, there is no information to suggest that the patient would have dystrophic calcinosis cutis from connective tissue disease. Additionally, the patient's age (very young) makes it unlikely that she would have any connective tissue disease except for potentially neonatal lupus erythematous, in which you would expect the patient to also have the characteristic annular pink-to-lilac plaques around the eyes and would also expect the patient's mother to have a positive ANA (additionally with most likely a positive anti-SSA/Ro antibody). Panniculitis can also cause dystrophic calcinosis, though these calcium deposits are usually widespread (rather than solitary), are extremely small, and are usually noticed on radiographic imaging rather than on clinical exam. A form of panniculitis, subcutaneous fat necrosis of the newborn, can be seen in infants; however, this most commonly occurs in term infants, usually as a response to hypothermia, hypoglycemia, or hypoxia. Sclerema neonatorum, on the other hand, is a panniculitis seen in severely-ill premature infants and is often fatal. Both would have been noticed during the patient's NICU stay as hard, indurated plaques and nodules on the back, buttocks, cheeks, shoulders, and thighs (subcutaneous fat necrosis of the newborn) or generalized sparing genitalia (sclerema neonatorum). Other causes of a more diffuse dystrophic calcinosis cutis include parasitic infection (unlikely in this situation as you would expect the patient to have a peripheral eosinophilia and widespread rather than solitary calcium deposits) and genetic syndromes such as EhlersDanlos and pseudoxanthoma elasticum (both unlikely given that the patient has only a solitary lesion of calcinosis and does not have any skin features that would suggest these genoderms at this time).What is the underlying pathophysiology behind metestatic calcinosis? - name some causesMetastatic calcinosis cutis: Due to abnormal calcium and phosphate metabolism; patient will have elevated calcium levels and/or phosphate levels Chronic kidney disease (secondary hyperparathyroidism) Milk alkali syndrome Sarcoidosis (secondary to increased calcium absorption from granulomas releasing 1,25- dihydroxyvitamin D) Ingestion of excess vitamin D supplements Neoplasms (hematologic or squamous cell carcinoma of head/neck) Tumoral calcinosis (familial) Idiopathic calcinosis cutis: Scrotal calcifications Milia-like calcinosis Subepidermal calcified nodule Tumoral calcinosis (sporadic) Iatrogenic calcinosis cutis: Extravasation of calcium-containing intravenous solutions Calcium-containing (alginate) dressings or calcium-containing electrodes Liver transplantation most commonly Gadolinium exposure in nephrogenic systemic fibrosis Mixed calcification: Combination of metastatic and dystrophic calcification which is a result of baseline calcium dysregulation which is then triggered by traumaRothmund=thomson syndrome - characterized by what triad - inheritance and gene - increased risk for what cutaneous malignancyRothmund-Thomson syndrome (RTS) is a very rare genodermatosis which is characterized by poikiloderma, alopecia, and osteosarcoma. RTS is extremely rare, with only around 300 cases reported in the literature; however, it has many dermatologic features which makes it relevant boards fodder. RTS is classified as one of the photodermatologic conditions due to the increased cutaneous photodamage observed. RTS is due to a RECQL4 helicase gene mutation and has an autosomal recessive (AR) inheritance. Poikiloderma (atrophy with hyperpigmentation, hypopigmentation, and telangiectasias) occurs at an early age with minimal sun exposure. This is most predominant on the face and is preceded by erythema and swelling at around age one which eventually transforms into poikiloderma. Erythema, swelling, and poikiloderma spreads to the extremities later but spares the trunk. Alopecia occurs early in life and presents with characteristically sparse/thin scalp hair with partial/full loss of eyebrows and eyelashes. In addition to these clinical features, patients are also of very short stature. Early-onset cataracts affect many of these patients and is another characteristic manifestation. Hypodontia and hypogonadism can also occur. Osteosarcoma is the most specific associated tumor, but patients are also at risk of cutaneous squamous cell carcinomas (SCCs). All patients with osteosarcoma require additional clinical evaluation for RTS, including a detailed family history. In addition to osteosarcoma, there may be numerous other skeletal abnormalities, including delayed bone formation, radial ray defects, metaphyseal chondrodysplasia, saddle nose, and frontal bossing.NF1 - skin findings - eye findings - diagnostic criteria - what is the size cut offs for CALMs - adults should have annual exams for what?Neurofibromatosis type 1 is an autosomal dominant (AD) condition. CALMs are congenital and typically appear in the first couple years of life. Axillary freckling, Lisch nodules, and optic gliomas usually become apparent between ages 3-10. Neurofibromas are more common around puberty. Plexiform neurofibromas and skeletal defects are likely congenital but many not manifest until later in life. To meet diagnostic criteria for NF1, two or more of the following must be present: 1. Six or more CALMs (>5mm in prepubescent individuals and >15mm in post pubertal individuals) 2. Two or more neurofibromas(option D.) of any type or one plexiform neurofibroma (option A.) 3. Inguinal or axillary freckling (option B.) 4. Optic glioma(s) 5. Two or more Lisch nodules(iris hamartoma) (option C.) 6. Osseous lesion (sphenoid wing dysplasia, thinning of long bone cortex) 7. First degree relative with NF1 At the time of diagnosis and annually during childhood, the following are indicated (Gutmann et al. JAMA 1997): Full skin exam, blood pressure monitoring for hypertension, evaluation of head circumference, height, and weight, evaluation of skeletal abnormalities, ophthalmologic examination, evaluation of precocious puberty, neuro-cognitive development, and school performance, and evaluation of plexiform neurofibromas. For adults, annual evaluation should include full body skin exam, PET/CT symptomatic lesions or defects, neurological examinations, blood pressure, and breast cancer screening in females.A 17-year-old male with a hereditary syndrome is presented in lymphoma tumor board. The patient's condition includes short stature, moderate immunodeficiency, low birth-weight, infertility, chronic obstructive lung disease, diabetes, and learning disabilities. Dermatologic symptoms include a history of photosensitivity with an erythematous, telangiectatic, and scaling rash in sun exposed skin and cafe-au-lait spots, telangiectasias, and hypo- and hyperpigmented areas. The patient has a birdlike facies with a prominent beaked nose, large protruding ears, long limbs, and large hands and feet. Which of the following is the most likely diagnosis?Bloom syndrome, named after dermatologist Dr. David Bloom, is a rare autosomal recessive (AR) disorder caused by mutation in the BLM gene. The disorder is characterized by genomic instability and patients can present with short stature, immunodeficiency, low birth-weight, infertility, chronic obstructive lung disease, diabetes, learning disabilities, and high-pitched voiced. Patients have a shortened life expectancy and the average life-span is 27 years. Dermatologic symptoms can include sun-exposed rashes that are erythematous, telangiectatic, and scaling, cafe-au-lait spots,telangiectasias, and hypo- and hyperpigmented patches of skin. The BLM mutation results in defect in DNA helicase and patients become prone to malignancies such as lymphoma, leukemia, and gastrointestinal, cutaneous, and genitourinary tumors (Arora et al., Int J Dermatol 2014). From the Bloom Syndrome registry, skin cancer arose in the 2nd and 3rd decades of life (average 31 years old), with basal cell carcinoma being the most common. No melanomas were reported. It appears that if patients with Bloom syndrome live into their 30s, the risk increases.What is the first cutaneous sign of tuberous sclerosisHypopigmented macules (Ash Leaf Spots) Li-Fraumeni syndrome is a rare autosomal dominant (AD) condition caused by a mutation in p53 tumor suppressor gene. These patients have an elevated incidence of breast cancer, osteosarcoma, leukemia, and brain tumors are with presentation at an early onset. Despite p53 mutation being common in basal and squamous cell carcinoma, these carcinomas are not typical in Li-Fraumeni syndrome. The lifetime probability of cancer in Li-Fraumeni syndrome ranges from 80-100%. Whole body MRI is a useful radiation-free screening tool used to detect many Li-Fraumeni-spectrum tumorsAplasia cutis congenita (ACC) of the lower extremities + dominant dystrophic epidermolysis bullosa (EB) describes which of the following syndromes? Howel Evans - key features, Gene, increased risk of what malignancyBart Syndrome Howel-Evans palmoplantar keratoderma (PPK) (Tylosis-esophageal carcinoma) is autosomal dominant (AD) condition caused by TOC gene mutation leading to pressure related focal nontransgredient PPK, oral leukoplakia and increased risk of esophageal cancer typically manifesting in the 5th decade of life. (Lee et al. N Engl J Med 2012) There are numerous types of palmoplantar keratoderma, all with varying gene mutations and clinical features.JXG - represents the most common what? - lesions typically occur in whom? where? presentation? - number one site of involvement outside the skin? second most common site? - patients with multiple lesions should be screened with what? - what is the association of JXG and NF1JXG represents the most common non-Langerhans cell histiocytosis. Although the etiology remains unknown, these are suggested to occur secondary to a disordered macrophage response to injury. Lesions typically occur in infants and young children, with >75% presenting during the first year of life. JXGs favor the upper half of the body (head and neck > upper torso > upper extremities > lower extremities) and appear as pink to red-brown, dome-shaped papules with later evolution to a more yellow appearance. Depending on the stage of the lesion, JXGs can appear similar to other entities, including melanocytic tumors, Spitz nevi, benign fibrous histiocytomas, reticulohistiocytomas, granular cell tumors, and xanthomas. If needed, biopsy can be utilized to differentiate these lesions from more worrisome diagnoses, including Spitz nevi and melanocytic lesions. (Kundak S et al. Int J Dermatol 2021) Histology demonstrates a well-demarcated dense superficial dermal infiltrate of histiocytes. Histiocytes in early lesions usually have abundant eosinophilic cytoplasm, whereas later lesions demonstrate histiocytes with foamy lipid-laden cytoplasm. Touton giant cells are a characteristic finding and contain "wreath-like" arrangement of nuclei at the periphery The most common site of extracutaneous involvementis the eye, namely the iris, occurring in <0.5% of patients. Hyphema and glaucoma are serious complications which can lead to blindness in some cases. Ocular involvement is more likely in patients with multiple cutaneous lesions. Current suggestions involve screening for eye involvement only in the presence of ocular or visual symptoms. The lung represents the second most common extracutaneous site of involvement. Other organ system involvement is rare but has been reported. In contrast to ocular screening, screening for visceral lesions in patients with multiple cutaneous lesions is suggested. (Samuelov L et al. Pediatr Dermatol 2018) Unlike other xanthomatous disorders, JXG is not associated with metabolic abnormalities, hyperlipidemia or diabetes insipidus. A rare, but known triple association involves JXG, neurofibromatosis type 1, and juvenile myelomonocytic leukemia. Findings suggestive of neurofibromatosis in conjunction with JXGs should warrant screening for juvenile myelomonocytic leukemia.Pachydermoperiostosis - cutaneous findings, most common findingsThe clinical photo demonstrates significant digital clubbing which involves all digits in a patient who started experiencing these changes during adolescence, along with arthralgias. In addition, he has a first-degree relative who shares his symptoms. This is consistent with pachydermoperiostosis (also known as primary hypertrophic osteoarthropathy). The most common cutaneous findings in patients with pachydermoperiostosis are digital clubbing (94%), and pachydermia (45%). (Castori et al., Clin Genet 2005) While all the above are findings may be seen in pachydermoperiostosis, the most common finding (other than digital clubbing) is pachydermia (D), which refers to progressive thickening and subsequent furrowing of the skin on the face and scalp.Bazex dupre christal - inheritance, gene - clinical features - what is Bazex syndromeThe dermoscopy photograph reveals a pearly, pink papule with arborizing vessels consistent with basal cell carcinoma (BCC). The median age of development of BCC is 68 years, and any BCC in a young individual should raise concern for an underlying condition. BDC is an X-linked dominant (XLD) disorder with the affected gene not yet identified but linked to Xq24-q27. Clinical features include follicular atrophoderma favoring the dorsal hands, milia, multiple BCCs (often on the face and can resemble melanocytic nevi), hypotrichosis, and localized hypohidrosis. In some observations, males tend to have a more severe phenotype and more hypotrichosis. BDC should not be confused with the similarly named condition Bazex syndrome, also known as acrokeratosis paraneoplastica, which is characterized by acquired keratotic plaques of the hands, feet, nose, ears, and cheeks in the setting of aerodigestive neoplasms. The plaques are often psoriasiform in appearance.Von hippel lindau syndrome - mutation in what gene, what does this gene do? what chromosome is it found on - key findings - what is the most common tumor in these patients?Von Hippel-Lindau syndrome (VHL) is caused by a mutation of the tumor suppressor gene, VHL, on chromosome 3p25 (Latif et al., Science 1993). This syndrome is inherited in an autosomal dominant fashion and causes both benign and malignant tumors. Pathology found in VHL include: Capillary malformations Renal cell carcinoma (clear cell variant) and renal cyts Hemangioblastomas of the brain, spine, and bilateral retinal capillaries Pheochromocytomas Endolymphatic sac tumors of the middle ear Pancreatic serous cystadenomas and neuroendocrine tumors Papillary cystadenomas of the epididymis and broad ligament Pulmonary and liver hemangiomas (McGrath et al. Clin Radiol. 1992) (Takahasi et al. Intern Med 2006) Clinical Pearl: Von Hippel-Lindau (VHL) syndrome is an autosomal dominant disorder caused by a mutation of the VHL tumor suppressor gene on chromosome 3. Patients can present with capillary malformations and benign and malignant tumors. Hemangioblastomas is the most common tumor in patients with VHL (60 to 84% of patients) and can occur in the brain, spine, or retinal capillaries.Horner Trantas dots - what are they and what do they indicate? - when do these occurHorner-Trantas dots are gelatinous hyperplastic papules involving the limbus, which indicate a diagnosis of atopic keratoconjunctivitis. This occurs in patients with atopic dermatitis, especially in the setting of dupilumab. Atopic dermatitis (AD) is a common dermatologic condition characterized by pruritus (required for diagnostic criteria) and a chronic relapsing rash which is associated with other atopic conditions, such as asthma, seasonal allergies, and food allergies. There are many clinical manifestations, including hertoghe sign (alopecia of lateral eyebrows), Dennie-Morgan lines (inferior eyelid linear creases), and xerosis, among others. There are several notable complications of AD, such as eczema herpeticum, eczema coxsackium, impetiginization, and molluscum contagiosum. Ocular complications can also occur (20-42% of AD patients). One of the most serious ocular complications of AD is atopic keratoconjunctivitis (AKC). This is an important entity, as it can lead to serious complications. As the name implies, AKC is found in the setting of AD. Recent literature has indicated dupilumab as an independent risk factor. This is important because dupilumab causes many forms of benign conjunctivitis, and it is imperative to distinguish these entities due to their drastic differences in prognosis and treatment. Patients with AD, not on dupilumab, often develop AKC later in the disease process (teens onward). Examination of tears has found elevated levels of activated eosinophils, mast cells, basophils, and T cells in patients with AKC, likely indicating their involvement in the pathophysiology. This is a non-infectious, inflammatory reaction. Non-specific symptoms, such as tearing, itching, redness, and eye burning, are common. Warning signs for AKC include pain, blurred vision, and persistent symptoms. Patients usually have preceding conjunctivitis. The tarsal conjunctiva has papillary hypertrophy. Limbal Horner-Trantas dots, as described in this question, are gelatinous hyperplastic papules involving the limbus. For test taking purposes, this is pathognomonic for AKC. Complications of AKC: Blindness Tear dysfunction Cataracts Steroid-induced glaucoma (from treatment) BlepharitisPachyonychia congenita - key findings - associated with multiple what? - genes involved - steatocystomas and neonatal teeth are most common in which typePachyonychia congenita (PC) is an uncommon autosomal dominant(AD) disease which affects keratinization predominantly ofthe skin and nails, and is associated with steatocystomas. Patients tend to present with a triad of symptoms which include (Shah et al. JAMA Dermatol 2014): 1. Plantar pain 2. Hypertrophic nail dystrophy (prominent subungual hyperkeratosis giving a triangular shaped nail) 3. Plantar keratoderma with underlying blisters Patients may also demonstrate follicular hyperkeratosis, cutaneous cysts (steatocystomas) and oral leukokeratosis. This condition is caused by a mutation in one of the following keratin genes (Wilson et al. Br J Dermaol 2014): KRT6A, KRT6B, KRT6C, KRT16, KRT17. In the past, PC was divided into two subtypes: Type 1 PC - (Jadassohn-Lewandowsky type) mutations in keratin 6a/16and Type 2 PC (Jackson-Lawler type) - mutations in keratin 6b/17. This disease is now stratified according to molecular genetic features (McLean et al. J Invest Dermatol 2011) with the associated mutation and classification demonstrated below: 1. KRT6A mutation: PC-K6a 2. KRT6B mutation: PC-K6b 3. KRT6C mutation: PC-K6c 4. KRT16 mutation: PC-K16 5. KRT17 mutation: PC-K17 Steatocystoma multiplex and neonatalteeth are most commonly seen in PC-K17 Patients typically present with hyperkeratosis of the finger/toenails in the first first years of life, with plantar/palmar keratoderma developing when weight-bearing begins with associated blisters, which is subsequently followed by chronic plantar pain. Management of this condition involves addressing the complications from nail thickening/dystrophy, cutaneous cysts, hyperhidrosis (botulinum toxin-A), and oral retinoids for the management of palmoplantar keratoderma. As an aside, topical simvastatin (down regulation of K6a promoter activity) is currently being tested in clinical trials for this condition. Olmsted syndrome, (mutilatiWhat are the classic eye findings of down syndrome? How about PXE, Riley Day syndrome, nail patella syndrome, hay wellsThe classic eye findings in Down syndrome (Trisomy 21), are Brushfield spots, epicanthic folds, upslanting palpebral fissures, fine lens opacities, and strabismus. Brushfield spots are pathognomonic for Down syndrome and are white, gray, or brown spots spaced around the periphery of the iris due to iris stromal connective tissue hyperplasia. (da Cunha et al. Am J Ophthalmol 1996) The classic eye findings in pseudoxanthoma elasticum (ABCC6 gene) are angioid streaks. Angioid streaks represent small breaks in Bruch's membrane which is an elastic tissue containing membrane of the retina. Patients with PXE can also develop macular degeneration, retinal pigment alterations, retinal hemorrhage, and blindness. The classic eye finding in Riley-Day syndrome is decreased corneal sensation to tear flow. Measurement of corneal tactile sensitivity evaluates the ophthalmic branch of the trigeminal nerve (CNV). The classic eye finding in Nail-Patella syndrome is Lester iris (zone of darker pigmentation around the papillary margin of the iris). The classic eye finding in Hay-Wells syndrome (AEC syndrome; Ankyloblepharon filiforme adenatum-Ectodermal dysplasia-Cleft palate syndrome) is ankyloblepharon (eyelid fusion by strands of skin). Patients can also have lacrimal duct atresia, lacrimal duct obstruction, conjunctivitis, and blepharitis.incontinentia pigmenti - gene involved, pathopysiology - disease is lethal in whom? most cases are due to what? - describe the presentationThis patient has incontinentia pigmenti (IP), which is caused by mutations in IKBKG, which encodes NF-κB essential modulator (NEMO). As its name implies, NEMO is involved in activating NF-κB, a transcription factor that serves to protect cells against apoptotic stimulation by TNF-α. The gene is located on the X chromosome, which makes the mutation lethal in XY males and also contributes to the Blaschkoid distribution of cutaneous findings due to lyonization (random inactivation of a particular copy of the patient's X chromosomes). Although inherited as an XLD condition, most cases occur due to sporadic mutations. IP typically presents soon after birth with linear, whorled, inflammatory vesiculobullous papules and plaques in a Blaschkoid distribution that then, with age, classically progress through verrucous, hyperpigmented, and hypopigmented/atrophic stages. Other ectodermal abnormalities are common, including whirled/linear alopecia of the scalp vertex, nail dystrophy, and dental abnormalities. Less commonly, patients have ocular and neurologic involvementdyskeratosis congenita is associated with what malignancies? In patients with ataxia telangiectasia, where do telangiectasias first appear? Verruciform Xanthomas can be a part of what genodermatosis?SCC ofthe oral mucosa and AML the bulbar conjunctiva CHILD SyndromeVon hippel lindau - gene, inheritance, - most common cutaneous findings - most common eye finding - name some other tumorsThe patient in the associated vignette demonstrates a presentation consistent with Von Hippel-Lindau syndrome (VHL). VHL is an autosomal dominant (AD) disorder caused by a mutation in the VHL tumor suppressor gene on chromosome 3p25 (Latif et al., Science 1993). The VHL gene product is a ubiquitin ligase involved in the regulation of a protein called hypoxia inducible factor 1-a (H1F1a). When VHL is mutated, H1F1a is uninhibited. H1F1a activates the transcription of numerous genes including endothelial growth factor, plateletderived growth factor B, erythropoietin, and other genes involved in glucose metabolism. (Wofford et al. J Am Acad Dermatol 2016) VHL syndrome is a progressive disease that is typically fatal by the 4 decade (metastatic renal cell carcinoma, growth of CNS hemangioblastomas). Skin findings are not common and typically affect <5% of patients with capillary malformations affecting the head and neck and café au lait macules being the most common cutaneous finding. The classic eye finding in VHL syndrome is retinal capillary hemangioblastoma (vascular tumor involving the retina with dilated, tortuous afferent and efferent vessels). Various tumors can occur and include but are not limited to: renal cysts, renal-cell carcinoma (clear cell variant), pancreatic cystadenomas and neuroendocrine tumors, pheochromocytoma, adrenal carcinoma, papillary cystadenomas of the epididymis and broad ligament, hemangioblastomas (cerebellar, medullary, spinal cord), retinal capillary hemangioblastoma. Tumor mass effects can lead to secondary complications including signs of increased intracranial pressure and spinal cord compression. In addition, cerebellar hemangioblastomas and renal cell carcinomas can produce erythropoietin leading to polycythemia. Surveillance screenings for VHL syndrome include the following: CT/MRI (brain, spinal cord, and abdomen), CBC, urinary vanillylmandelic acid level, serum catecholamine levels. Patients should be referred to the appropriate surgical specialists for management/removal of tumors. Patients should also be referred to ophthalmology for management of retinal hemangioblastomas as these lesions can lead to visual impairment and blindness.name the cutaneous findings in ehlers danlons syndrome? Arthrochalasia type is caused by a defect in what? presents with what?Ehlers-Danlos syndrome is a heterogenous group of disorders caused by various collagen, collagen enzyme, or collagen associated protein defects. Typical cutaneous features include: Hyperextensible and fragile skin Easy bruising Cigarette-paper-like scarring Gaping/fish mouth scars Hypermobile joints Hernias Blue sclerae Gorlin's sign (ability to touch tongue to tip of nose. Seen in 50% of Ehlers-Danlos and 10% of normal population) Absence of lingual frenulum Type VIIA and VIIB Ehlers-Danlos (Arthrochalasia type) is caused by a genetic defectin COL1A1, COL1A2 leading to defective alpha1 and alpha2 chains oftype I collagen. This leads to hyperextensible skin and joints with severe congenital hip dislocations and scoliosis.Classic type ehlers danlos is caused by a defect in what? Vascular ehlers danlos is caused by a defect in what? Progeroid type is caused by a defect in what?Type I and II Ehlers-Danlos (Classic type) is caused by a genetic defect in COL5A1, COL5A2 leading to defective alpha1 and alpha2 chains of type V collagen. This leads to hyperextensible joints, skin, fish mouth and atrophic scarring, easy bruising, and absence of inferior lingual and labial frenulum with positive Gorlin's sign. Type IV Ehlers-Danlos (Vascular type) is caused by a genetic defect in COL3A1 leading to defective alpha1 chain of type III collagen. This causes thin, translucent skin with visible venous networks, excessive bruising, joint hypermobility, distinct facies, and risk of arterial, uterine, GI, and other visceral rupture with risk of early death. Ehlers-Danlos (Progeroid type) is caused by a genetic defect in B4GALT7 leading to defective galactosyl transferase-1. Skin is hyperextensible and joints are hypermobile. In addition, patients have osteopenia, growth retardation and progeroid facial features."Cherry Red Spot" Is seen in what condition? cause of this condition? pathophysiology Most common defect in muir torreThe photograph depicts a cherry-red spot due to the accumulation of abnormal protein in the ganglion cells surrounding the fovea. (Chen et al. Surv Ophthalmol 2014) (Aragao et al. Arg Bras Oftalmol 2009) This ocular finding, combined with the systemic features, is typical of Tay-Sachs disease (a lysosomal storage disorder) which is caused by a defect in the hexosaminidase A enzyme. This enzyme defect leads to the accumulation of ganglioside GM2 which is deposited throughout the body including the brain and retina. MSH2 is the most common mutation in muir torreLoeys Dietz syndrome - inheritance, genes - shares features with what conditions - what are the distinctive characteristics? - what feature distinguishes it from marphans?The photos associated with this question demonstrate hypertelorism, translucent skin, easy bruising, and facial asymmetry from a previous stroke. The clinical findings, genetic abnormality, and patient history are diagnostic of Loeys-Dietz syndrome. Loeys-Dietz syndrome is an autosomal dominant (AD) connective tissue disorder secondary to mutations in TGFbR1, TGFbR2, SMAD3, or TGFb2 genes. Loeys-Dietz syndrome shares features with Marfan syndrome and Ehlers-Danlos syndrome but has other characteristic features that help distinguish it from these conditions. LoeysDietz syndrome is characterized by: (Loeys et al. Nat Genet 2005) Aortic aneurysms, Generalized arterial tortuosity, Hypertelorism Bifid/broad uvula, Translucent skin Skeletal anomalies There is a spectrum of disease which may include other cardiovascular abnormalities, allergies, eczema, GI symptoms (constipation, eosinophilic GI disease, IBD), craniofacial abnormalities (uvula anomalies, palate anomalies, dental enamel defects), learning disabilities, visceral rupture, headaches, blue sclera, retinal detachment, pneumothoraces, etc. Skeletal features in Loeys-Dietz syndrome may overlap with Marfan syndrome and includes pectus deformity, scoliosis, and pes planus. Patients with Loeys-Dietz syndrome have more widespread and aggressive vascular disease when compared to Marfan syndrome. There is lack of lens dislocations in Loeys-Dietz syndrome which can also help distinguish this condition from Marfan syndrome.PLS - what is it? -PLS (palmoplantar keratoderma with periodontosis) is a hereditary palmoplantar keratoderma (PPK) with autosomal recessive (AR) inheritance. Palmoplantar keratodermas include a group of hereditary and acquired disorders of cornification that are characterized by marked hyperkeratosis of the palms and soldes. Hereditary PPKs often include a constellation of associated extracutaneous manifestations. Key features of PLS: Skin Sharply demarcated palmoplantar keratoderma with erythematous border. Transgradiens PPK noted (extension of hyperkeratosis to dorsal surface of hands and feet, achilles tendon). Soles typically have greater involvement than palms. Keratoderma onset can be as early as the first three months of life, but more commonly occurs between 1 and 4 years of age, with simultaneous onset of periodontitis. Pyogenic infection Some reports suggest that PLS patients are at an increased risk fo cutaneous bacterial and fungal infections, as well as systemic infections, such as hepatic abscesses. Hyperkeratotic plaques on elbows and knees Hyperhidrosis and malodor of hands and feet Hair May be sparse Mouth Patients with PLS present with palmoplantar keratoderma and destructive periodontitis, beginning in early childhood leading to premature loss of teeth Central nervous system Dural calcification at the tentorium and choroid attachments. Diffuse dural calcifications PLS is associated with mutations in the gene encoding cathepsin C. Note: diffuse dural calcification can also be seen in basal cell nevus syndrome (BCNS), which is caused by autosomal dominant mutations in the PTCH gene, which encodes the patched tumor suppressor protein of the sonic hedgehog signaling pathway. Treatment: It is important that patient has close follow-up with dermatology and dentistry. Brain CT/MRI scans should be considered to rule out CNS findings as above, with evaluation of neurology. It is important that patients practice meticulous oral hygiene, regular plaque removal. Oral retinoids are used to improve hyperkeratosis and decrease risk of periodontal complications. Topical keratolytics are also useful for improving keratoderma. PLS patients are reported to experience persistence persistent keratoderma throughout their lives, though periodonitis tends to resolve after all teeth are lost.The variant of basal cell nevus syndrome cause by the SUFU mutation has an increased risk of what and a decreased risk of what?The correct answer is D. The variant of BCNS is caused by mutations in SUFU, and has an increased risk of medulloblastoma and a decreased risk of odontogenic keratocysts. BCNS is most often caused by a mutation in PTCH1, which is a receptor in the sonic hedgehog homolog (shh) pathway. In the absence of the shh ligand, PTCH inhibits the protein smoothened (SMO), preventing downstream signaling. However, some cases are caused by inactivating mutations in SUFU, which ordinarily represses the activity of Gli and beta-catenin transcription factors. Patients with mutations in SUFU have an increased risk of medulloblastoma (particularly the desmoplastic variant), and thus require MRI every 4 months until age 3, and then twice a year until age 5. For children with PTCH1 mutations, a single MRI at diagnosis should be considered with appropriate symptoms (abnormal neurological development, for example). Odontogenic keratocysts are the second most common finding in BCNS (after BCC), but are less common with SUFU mutation. Thus, for patients with PTCH1 mutations, screening should be done at diagnosis and once every two years until age 22 (if there are no anomalies), but for patients with SUFU mutations there are no clear recommendations.Spiny Keratoderma - inherited vs acquired - Acquired is associated with what?Spiny keratoderma is a rare condition with unknown etiology. One study found keratins 6 and 16 (markers of hyperproliferation) were overexpressed in the keratotic projections. Another study showed that AE13, an antibody specific to keratin expressed in normal hair cortex was positive in the keratotic lesions and that keratinization was occurring without the production of keratohyalin granules (a feature of the normal hair cortex). Based on these findings, the authors concluded that spiny keratoderma represents aberrant ectopic hair. Another hypothesis is that coenzyme A reductase inhibitors may disrupt lamellar bodies and cholesterol synthesis resulting in epidermal hyperplasia. Spiny keratoderma can be hereditary (autosomal dominant) or acquired. When inherited, it presents at a younger age, and although cosmetically and physically bothersome to some, it is benign. When acquired, it presents in older adults (typically after age 50) and is associated with a variety of systemic diseases and malignancies. In patients presenting with acquired spiny keratoderma, further evaluation for malignancy or systemic disease is warranted. Acquired spiny keratoderma associations include: Malignancy Rectal CA Bronchial CA Renal CA Breast CA Leukemia Squamous cell carcinoma of the skin Melanoma Systemic diseases Darier disease Type IV hyperlipoproteinemia Chronic renal failure Pulmonary TB Lipid abnormalities Asthma Myelofibrosis Polycystic kidney disease with liver cysts Type II diabetes Clinically, lesions present as multiple small firmly-attached keratotic spiny spicules/projections isolated to the palms and soles. The spines resemble an old-fashioned music box, and because of this, spiny keratoderma is sometimes referred to as "music box" spiny dermatosis. Other less favored terms have included punctate porokeratotic keratoderma, palmoplantar filiform hyperkeratosis, and minute digitate hyperkeratosis. Biopsy shows acral skin with denser orthokeratotic keratin with intervening narrow columns of parakeratosis that arise from the inter-adnexal epidermis. Underlying the areas of parakeratotic epidermis there is an attenuated granular layer (hypogranular) but without dyskeratosis or vacuolar changes. Treatments options, although unsatisfactory include: mechanical debridement, topical emollients, keratolytics, Carmol 20, topical retinoids, acitretin, calcipotriene, topical vitamin D analogues (tacalcitol, calcipotriene), urea, 5-FUCarvajal syndrome - cause of disease - characteristic featuresCarvajal syndrome is an autosomal recessive (AR) genodermatosis caused by a defect in the desmosomal protein, desmoplakin, which is letter A of the associated image. Clinical features include the following: Woolly hair Striate palmoplantar keratoderma (PPK), predominantly at sites of pressure Left sided arrhythmogenic cardiomyopathy Follicular keratoses Digital clubbing (secondary to heart failure). Early recognition of this disorder and appropriate intervention with implanted automatic cardioverter defibrillators and use of antiarrhythmic medications can prevent sudden cardiac death in these patients in whom left ventricular dilatation may be asymptomatic at a young age. (Srinivas et al. Int J Trichology 2016) Naxos syndrome may be a variant of Carvajal syndrome and is caused by an autosomal recessive (AR) gene defect which encodes the desmosomal protein, plakoglobin. Clinical features include the following: Diffuse PPK Wooly hair Arrythmogenic right sided cardiomyopathyName syndromes associated with the following mutations: - connexin 26 - connexin 30 and 31 - desmoglein 1 - desmocollinGap junctions are made up of connexons. Mutations in connexin proteins cause disorders including: Connexin 26 (Vohwinkel's syndrome, KID syndrome, PPK with deafness) Connexin 30.3, 31 (Erythrokeratoderma variabilis) Connexin 30 (Hidrotic ectodermal dysplasia, aka Clouston syndrome) An autosomal dominant (AD) genetic defect in desmoglein 1 causes striate PPK (PPK type 1) Autoantibodies against desmoglein: Pemphigus foliaceus and staph scalded skin syndrome Autoantibodies against desmoglein 1and 3: Mucocutaneous pemphigus vulgaris Autoantibodies against desmoglein 3: Mucosal pemphigus vulgaris Autoantibodies against desmocollin 1: IgA pemphigus (Sneddon-Wilkinson)Hyper IgE syndrome - gene - characteristic features - treatment for these patients usually includes what?Hyper-immunoglobulin E syndrome (HIES) presents as either the classic autosomal dominant form (AD-HIES) due to a mutation in STAT3, as well as an autosomal recessive (AR) form due to DOCK8 mutations. Both are characterized by recurrent skin and sinopulmonary tract infections, early-onset eczematous dermatitis, and high IgE levels. The question stem here describes AD-HIES, which has skin infections that are usually caused by Staphylococcus aureus, or cold abscesses, development of coarse facies by adolescence, as well as scoliosis and retained primary teeth. The mainstay of therapy for AD-HIES is antistaphylococcal antibiotics, and so patients are usually treated prophylactically with trimethoprim-sulfamethoxazole.What are the next steps in management of a suspected tuberous sclerosis patient - referral to whom? - should a biopsy be done?This question is asking about management of a newly diagnosed tuberous sclerosis patient. A thorough review of systems is the first step, as this will allow for detection of disease manifestations and complications that are potentially treatable. Referralto pediatric cardiology and neurology is essential to detect brain lesions, seizures, heart lesions and arrhythmias. Referral to pediatric gastroenterology for abdominal MRI and pulmonology for lung imaging and pulmonary function testing (PFTs) can also be considered, although these are not listed as answer choices. Punch biopsy of a facial angiofibroma is generally not needed to make a diagnosis of TSC. While excision of periungal fibromas is a possibility, and may be considered for lesions that are painful, this would not be one of the 3 best next steps. Referral to a pediatric geneticist is reasonable, but is less important than the three correct answers. Detection of disease manifestations and treatment should not be delayed while waiting on genetic testing results for a very clear clinical diagnosis. Topical rapamycin has anecdotal evidence of success in treating angiofibromas and periungual fibromas in TSC, but would not be an initial best step. Vismodegib is a smoothened inhibitor used in patients with basal cell nevus syndrome and unresectable basal cell cancers.Wiskott aldrich - inheritance, gene - key features - lab findings (what do the immunoglobulin levels look like) - TreatmentWiskott-Aldrich (Eczema-thrombocytopenia-immunodeficiency syndrome) is an X-linked recessive immunodeficiency disease caused by a mutation in the WASP gene which demonstrates atopic-like dermatitis, recurrent sinopulmonary infections, and petechiae/ecchymosis due to microthrombocytopenia. Patients are susceptible to bacterial and viral infections, pneumocystis, meningitis, sepsis, and early death (average survival is age 15). The causes of death are infection, bleeding, and lymphoreticular malignancy for which these patients are at an increased risk (~25%). Recurrent bacterial infections typically start following diminishing levels of placentally transferred maternal antibodies around 3 months of age. Laboratory findings include persistent thrombocytopenia, lymphopenia, eosinophilia, and low IgM and IgG2 levels. IgA, IgE, and IgD tend to be elevated. Treatment is with hematopoietic stem cell transplantation. Prophylactic antibiotics and antivirals can decrease the risk of fatal infections. IVIg can help prevent infection and topical steroids can be used for dermatitis.Hyper IgM syndrome - deficiency in what? - key features Hyper IgD syndrome - key features IgA deficiency - most common what? - associated with anaphylaxis to what?Hyper IgM syndrome - Deficiency in CD40 ligand. Key features include extensive warts, oral ulcers and diarrhea. Laboratory findings include elevated IgM, low platelets and neutrophils. Hyper IgD syndromes are part of a spectrum of diseases characterized by episodes of fever, arthritis, myalgias and skin rash. These episodes last for days and recur periodically throughout life. Isolated IgA deficiency is the most common immunodeficiency (1 in 600). Most patients are well, but may manifest with atopy (allergy, asthma and atopic dermatitis) in addition to possible autoimmune disease. One key testable factoid is the associated anaphylaxis with IVIG and other transfusionsPapillon lefevre syndrome - gene, what does this encode for - key features - What features seperate Haim Munk Syndrome from papillon lefevre?Papillon-Lefèvre syndrome is a hereditary palmoplantar keratoderma (PPK) with autosomal recessive (AR) inheritance involving a mutation in CTSC, encoding cathepsin C. Haim-Munk syndrome is an allelic condition with similar clinical features and is also associated with CTSC mutation,transgrediens PPK, and periodontitis with tooth loss. Of the group of PPKs, only Papillon-Lefèvre syndrome and Haim-Munk syndrome classically involve premature destructive periodontitis with tooth loss. The key features of Papillon-Lefèvre syndrome include: Sharpy demarcated transgrediens PPK of the hands and feet (extension of hyperkeratosis to dorsal hands and feet) with erythematous border, hyperhidrosis, and malodor) Pyogenic infections Hyperkeratotic psoriasiform plaques on elbows and knees Periodontitis with tooth loss Dural calcifications at the tentorium The PPK usually presents between 1-4 years of age, along with periodontitis. Along with close follow-up with dermatology and neurology, patients should also be seen frequently by dentistry for treatment of severe oral disease. In addition to the above characteristics, Haim-Munk Syndrome also features findings including arachnodactyly, acro-osteolysis, atrophic nail changes, and finger deformities.Those with BAP1 germline mutation have an increased risk of what malignancies? - how does a BAPoma present - these patients should have yearly screening by whom?Those with BRCA-1 associated protein 1 (BAP1) inactivated melanocytic tumors (BIMT) or BAPomas with an underlying germline mutation have a greater risk of uveal melanoma, breast cancer, and thyroid cancer. These neoplasms often present in childhood/adolescence, and clinically resemble fleshy pink to brown nevi. Although previously thought to be pre-malignant and variants of Spitz nevi, current evidence does not support malignant potential of these neoplasms. There is not abundant evidence regarding management, although conservative re-excision for definitive margin analysis is often pursued. BAP1 is a tumor suppressor gene located on chromosome 3. Associated with BRCA-1, it plays a role in DNA repair. These tumors are characterized immunohistologically by loss of BAP-1 expression. Germline mutations have an autosomal dominant (AD) inheritance pattern, but spontaneous mutations have also been reported. From a dermatologic perspective, these patients are at risk for cutaneous and ocular/uveal melanoma. Given this risk, yearly screening with an ophthalmologist is recommended for these patients. Patients with germline mutations are also at risk for mesothelioma, renal cell carcinoma, and less so basal cell carcinoma, thyroid cancer, breast cancer, neuroendocrine tumors, meningiomas, and cholangiocarcinoma. For patients with biopsy-proven BIMTs/BAPomas, genetic screening is important to determine their risk for associated malignancies. Keratoacanthomas have been associated with Muir-Torre syndrome (MSH/MLH mutations), Gryzbowski syndrome, as well as others. They have not been reported with a greater incidence in those with BRCA-1 associated protein 1 germline mutations.Most likely initial presenting feature of NF1 A patient with a history of gastrointestinal polyposis presents with alopecia, generalized pigmentation, and nail dystrophy. Which of the following is the most likely diagnosis? A patient with multiple lentigines and blue nevi may also have:CALMS Crohkite canada syndrome Atrial myxoma as part of carney complexA 7-year-old female presents with non-painful thickened "rock-hard" skin which started on the left thigh but has since began involving the left buttock. Thickened skin was noticed during infancy but has progressively worsened. Skin overlying the thickened area appears normal. Histology reveals normal epidermis but mild dermal thickening, no inflammation, swollen bundles of collagen oriented horizontally, increased fibroblasts, and adipocyte entrapment. Her mother and maternal grandfather also have the same diagnosis. The pathogenesis of this skin condition involves increased signalling of which of the following?This patient has Stiff Skin syndrome (SSS) characterized by the bilateral, thick, hard, and progressive skin induration. Adipocyte entrapment and thick collagen bundles on biopsy confirm this. Various locations may be affected including buttocks, thighs, shoulder girdle, and legs, with involvement spreading to multiple locations with age. The family history suggests an autosomal dominant pattern of inheritance; the FBN1 gene leads to fibrillin 1 defect and subsequently increased signalling of TGF-β, leading to a scleroderma-like presentation with excess microfibrillar deposition and defective elastogenesis. Complications of this sclerosis include impaired mobility, restrictive lung disease, and joint contractures.A patient with multiple sebaceous adenomas should be screened with which of the following examinations? Benign familial pemphigus is inherited in what fashion? Patients that are diagnosed with Peutz-Jeghers syndrome must have a colonoscopy for hamartomatous polyps transforming to carcinoma every:Colonoscopy as part of muir torre syndrome autosomal dominant every three yearsName syndromes associated with multiple trichoepitheliomas Cutaneous osteomas are seen in what condition Halequin ichtyosis: what is uniformly absent? describe the keratohylin granules in each typeSeveral syndromes have been associated with multiple trichoepitheliomas: Bazex, Brooke-Fordyce, Brooke-Spiegler, Rombo Explanation: Albright hereditary osteodystrophy is caused by mutations in the Gs subunit of adenylate cyclase. There is calcification and ossification due to pseudohypoparathyroidism, absent 4th knuckle, and hypogonadism. Harlequin ichthyosis, the lamellar granules are uniformly abnormal or absent. On electron microscopy, there is no evidence of the lipid lamellae which form due to lamellar granule discharge into the region between the granular and cornified cells. This is suggestive of a primary defect in lipid synthesis and protein dephosphorylation resulting in faulty lamellar body formation and secretion. Harlequin fetus/ichthyosis can be divided into three types based on the microscopic appearance of the keratohyaline granules. Type I - normal, Type II - too small to be seen by light microscopy, Type III - absent keratohyaline granules.What is the condition associated with a connexin 30 mutation? A 6-year-old boy presents with brachyonychia and three firm subcutaneous nodules with a bluish hue on the trunk. Biopsy of a representative lesion is consistent with a pilomatricoma. What is the most likely associated syndrome? In chronic granulomatous disease, the diagnosis is made by which of the following tests?Clouston syndrome is associated with a defect in Connexin 30 (GJB6 gene). Findings include palmoplantar keratoderma with transgradiens, dystrophic nails, sparse hair with absent body, eyelash, and eyebrow hair after puberty. Rubenstein Taybi: (CREBBP)Abnormal faces, keloids, pilomatricomas Nitroblue tetrazolium testWhat is defective in naxos syndrome Most common skin change found in POEMS syndromePlakoglobin POEMS syndrome, also known as Crow-Fukase syndrome, is an acronym for polyneuropathy, organomegaly, endocrinopathy (most common being hypogonadism), M-protein (IgG and IgA light chains), and skin changes. Of the cutaneous signs, hyperpigmentation is the most common, occurring in up to 90% of affected patients. (can also see, skin thickening, hypertrichosis, sclerodermoid changes)A 20-year-old woman presents with hypodontia, sparse hair, palmoplantar keratoderma, and nail dystrophy. Examination of her eyelids reveal multiple, translucent-appearing papules. The most likely gene defect is: Low-cystine content in hair and nails may contribute to the phenotype seen in:Schopf-Schulz-Passarge syndrome is characterized by multiple apocrine hydrocystomas (most commonly appearing on the eyelids) and syringofibroadenomas, in addition to hypodontia, hypotrichosis, onychodystrophy, and palmoplantar keratoderma. It is inherited as an autosomal recessive condition and is caused by mutations in WNT10A, which encodes a cutaneous signaling molecule involved in ectodermal appendageal development. Tay syndrome is also known as trichothiodystrophy, or IBIDS: (photosensitivity), icthyosis, brittle hair, intellectual impairment, decreased fertility, and short stature. Hair shaft has a characteristic "tiger tail" appearance under polarized light and the low cystine content in hair and nails is thought to be responsible for the phenotype seen."Mousy" odor of urine is characteristic of which of the following syndromes LEMD3 is mutated in which of the following syndromes? What fact distinguishes type 1 and type 2 hereditary hemorrhagic telangectasiaphenylketonuria Buschke ollendorf Type I families have an increased incidence of pulmonary arteriovenous fistulasWhich of the following eye findings is caused by the rupture of Bruch's membrane? Apocrine hidrocystoma may be associated with which of the following syndromes? Keratins 1 and 10 are important intermediate filaments in suprabasal keratinocytes. Genetic mutation of these keratins can lead to which of the following disorders?Angioid streaks of pseudoxanthoma elasticum Schopf-Schulz-Passarge is a syndrome of ectodermal dysplasia which presents with keratosis palmoplantaris with hypodontia, hypotrichosis, and cysts of the eyelids. epidermolytic hyperkeratosisLate onset subungual keratotic tumors are associated with: Exposure to what medicine in-utero has been most closely associated with aplasia cutis congenita? Dystrophic epidermolysis bullosa is associated with mutations in collagen VII. Trauma or friction induced blistering in these patients have a plane a splitting in the:incontinentia pigmenti Methimazole Sublamina densaSpastic diplegia is associated with which of the following disorders? A teenage female presents with the complaint of "nail fungus". On exam, she has triangular lunulae, palmoplantar hyperhidrosis, micronychia and an absent patella. Which of the following gene defects is most likely in this patient?Sjogren-larsson Syndrome Nail-patella syndrome is an autosomal dominantly inherited condition due to mutations in LMX1B, which encodes a transcription factor that regulates collagen synthesis. The condition most frequently involves the thumb and may also involve the other fingers but to a lesser extent. The nails are absent or hypoplastic, and the dystrophy is usually more marked on the radial side of the digit. A triangle-shaped lunula is commonly observed. The nail changes are typically associated with bony abnormalities, including absent or hypoplastic patellae, radial head dysplasia, and iliac crest exostoses ("horns"). In children, the diagnosis is best confirmed by the presence of iliac horns on pelvic X-ray.Which of the following is the first symptom of ataxia telangiectasias? A double row of eyelashes is associated with: A 15 year old male presented with very dry, scaly skin. He notes that his grandfather and maternal uncles had similar findings but were otherwise healthy. His mother did have a prolonged labor when delivering him. Which of the following types of cancer may he be at increased risk of?Cerebellar ataxia lymphedema distichiasis syndrome: a form of congenital lymphedema Testicular, due to steroid sulfatase deficiencyPatients with Darier's disease are at increased risk for:Kaposi varicelliform eruption is the condition in which viral infection occurs in a patient with a pre-existing chronic dermatitis. Darier's disease is an autosomal dominant genodermatosis caused by a mutation in ATP2A2 which encodes SERCA2. Cutaneous manifestations of warty, hyperkeratotic papules in a seborrheic dermatitis, which may be infected with HSV or bacteria.A middle-aged woman complains of multiple rough lesions on her trunk. Biopsy reveals connective tissue nevi. Because other family members have had similar lesions, genetic testing is performed, revealing a defect in the LEMD3 gene. What is the classic bone finding in patients with a defective LEMD3 gene? A patient with thyroid carcinoma and cobblestone-like changes of the oral mucosa will also likely have: Which disease is found more commonly in mothers of patients with chronic granulomatous diseaseOsteopoikilosis (Buschke olendorf syndrome) Tricholemmomas Female carriers of chronic granulomatous disease have an increased incidence of discoid lupus, infections, and aphthous stomatitis.Lamellar ichthyosis is caused by a defect in transglutaminase 1. It can present as a colloidion baby at birth with subsequent large thick plates of scale especially on flexures, ectropion and eclabium. If two unaffected carrier parents have a child, how likely is their child to have this condition? Of the following, Cowden's disease is most commonly associated with this kind of malignancy: Which of the following is defective in Ehlers-Danlos syndrome (EDS) with congenital adrenal hyperplasia?Lamellar ichthyosis is an autosomal recessive condition. Follicular thyroid Tenascin XThe deficiency in familial multiple cutaneous leiomyomatosis is: A patient with coarse facies, broad nasal bridge, and extensive eczema might be expected to have which abnormal laboratory value? Pruritus in Sjogren-Larsson syndrome is attributed to accumulation of what molecule(s)?Fumarate Hydratase High IgE levels leukotrieneMutations in calcium transporters cause which pair of diseases? A 20-year old male with a history of pheochromocytoma and medullary thyroid cancer presents with mucosal papules. His overall body appearance is most likely to demonstrate: Mutations affecting the VEGF receptor-3 cause which of the following disorders?hailey hailey and darier disease Marphanoid Hereditary lymphedemadiagnostic criteria for NF1The 7 clinical criteria used to diagnose NF1 are as follows: Six or more café-au-lait spots or hyperpigmented macules greater than 5 mm in diameter in prepubertal children and greater than 15 mm postpubertal Axillary or inguinal freckles (>2) Two or more typical neurofibromas or one plexiform neurofibroma Optic nerve glioma Two or more iris hamartomas (Lisch nodules), often identified only through slit-lamp examination by an ophthalmologist Sphenoid dysplasia or typical long-bone abnormalities such as pseudarthrosis First-degree relative (eg, mother, father, sister, brother) with NF1You receive a hospital consult from the gastroenterology service for a 42-year old woman with esophageal cancer. They would like your opinion on the yellow, thickened areas on her palms and soles in areas of pressure. When you speak with her, she says that her father had similar problems and it runs in her family. Which gene is mutated? Patients with hemochromatosis are at increased risk for what things?Howel Evans syndrome: RHBDF2 gene Patients with hemochromatosis have increased intestinal iron absorption leading to systemic iron overload. Signs include a generalized metallic-grey to brown hyperpigmentation, koilonychia, alopecia (especially pubic/axillary hair), cardiac failure/arrhythmias/heart block, hepatomegaly with cirrhosis, diabetes (bronze diabetes), polyarthritis with chondrocalcinosis and are susceptible to Vibrio vulnificus and Yersinia infections.Adenosine deaminase deficiency is associated with what things? What phenotype results from a low activity of double stranded RNA adenosine deaminase?SCID dyschromatosis symmetrica hereditariaRichner Hanhart syndrome - inheritance, defect - pathophysiology - characterized by what two things?Richner-Hanhart syndrome (hereditary tyrosenemia type II) is an autosomal recessive disorder caused by a deficiency in hepatic tyrosine aminotransferase (TAT gene), the first enzyme in the tyrosine degradation pathway. As a result, plasma tyrosine and its metabolites are elevated. This disease is characterized by painful PPK and pseudoherpetic keratitis or ulcers, which can result in blindness. Treatment is low-tyrosine/phenylalanine diet; this approach can result in resolution of the skin and eye lesions.A patient with multiple facial trichilemmomas is at risk of which of the following cancers? What is the gene mutation that most commonly causes Herlitz junctional epidermolysis bullosa? Features of NF2Breast Cancer (Cowdens) LAMB3 (Laminin 332) Neurofibromatosis type II is an autosomal dominant disorder caused by mutations in schwannomin/merlin. Clinical features include cutaneous schwannomas and neurofibromas, bilateral vestibular schwannomas, and juvenile posterior subcapsular lenticular opacities. Optic gliomas are usually seen in NF1.CHIME syndrome Most common location for the keratotic plaques of KID syndrome? Which syndrome is characterized by hyperhidrosis, lack of pain sensation, hypersalivation, and absent fungiform papillae?CHIME syndrome has all the findings of colobomas of the eye, heart defects, ichthyosiform dermatosis, intellectual disability, and ear defects. Face Riley Day Syndrome (Familial dysautonomia)Findings of dysplastic nevi and melanoma inherited in an autosomal dominant fashion is linked with which of the following? A 16 month-old girl presents with patchy alopecia, a whorled and erythematous scaly eruption, and asymmetric limb shortening. What laboratory or radiologic test may aid in diagnosis?Pancreatic cancer The patient has Conradi-Hunermann Syndrome. This is a X-linked dominant disorder characterized by ichthyosiform erythroderma in Blaschko's lines in infancy which resolves with follicular atrophoderma, patchy alopecia, short stature, cataracts, scoliosis, assymetric limb shortening. Bone films will demonstrate stippled epiphyses. Ichthyosis and stippled epiphyses resolve after infancy.Lipoid proteinosis - inheritance, defect - characterized by what features, calcifications where? In biopsies from blisters in patients with junctional epidermolysis bullosa, the split is found in the: carriers for chronic granulomatous disease are at increased risk for what?Lipoid proteinosis is an autosomal recessive disorder caused by defects in the extracellular matrix protein 1 gene. It is characterized by scars and yellow papules of the face and oropharynx, yellowish papules on the eyelid margin, hoarse voice, verrucous nodules of the elbows and knees, and calcification of the temporal lobe and hippocampus with occasional seizures. lamina lucida Discoid lupusWhich of the following disorders is associated with delayed separation of the umbilical cord? What gene is responsible for NF-1 like syndrome?Leukocyte adhesion deficiency SPRED-1 acts as a suppressor of the Ras/MAPK pathway, the same pathway involved in neurofibromatosis-1. Likewise, defects in SPRED1 are the cause of neurofibromatosis type 1-like syndrome, also known as Legius Syndrome. Legius Syndrome has the following characteristics: cafe au lait spots, axillary and inguinal freckling, learning disability, and macrocephaly; likewise patients can actually meet criteria for the clinical diagnosis of NF-1. Although Legius Syndrome has many overlapping features with NF1, neurofibromas, lisch nodules, and CNS tumors have not been reportedRefusum syndrome - issues with what organ How is mafucci syndrome inherited? Cutaneous findings associated with Downs syndromePatients with Refusum syndrome have a deficiency in phytanic acid oxidase. The ocular abnormalities seen are pigmentary retinopathy, cataracts, nystagmus, and night blindness. Sporadic inheritence Down syndrome is caused by nondisjunction and results in trisomy 21. Clinical features include single palmar crease, flat nipples, increased nuchal skin folds, syringomas, elastosis perforans serpiginosa, xerosis, epicanthic folds of eyes, protruding scrotal tongue and fissured thickened lips.Erythropoietic protoporphyria - gene, presentation what are the genes for PCT, acute intermittent porphyria, and congenital erythropoietic porphyria, variegate prophyriaErythropoietic protoporphyria is a relatively common form of porphyria, characterized by photosensitivity. Over time, superficial waxy scars and milia are identified in sun-exposed areas. The defective gene is ferrochelatase.Uroporphyrinogen decarboxylase is mutated in porphyria cutanea tarda. Porphybilinogen deaminase is defective in acute intermittent porphyria. Uroporphyrinogen III synthase is defective in congenital erythropoietic porphyria. Protoporphyrinogen oxidase is mutated in variegate porphyria.A Puerto Rican woman is seen in clinic for a pruritic rash on her trunk. A punch biopsy is performed. The biopsy site continues to bleed, with hematoma formation. The bleeding is eventually controlled. On further exam, her skin and hair are light brown. She has a history of granulomatous colitis. What it the most likely reason she had excess bleeding with a simple procedure? Which of the following metals is deficient in the serum of patients with Menkes kinky hair syndrome? - inheritance, gene, presentationPatients with Hermansky-Pudlak syndrome have platelets without dense bodies, which results in excess bleeding. Other features of this condition include oculocutaneous albinism, ceroid lysosomal storage disease resulting in pulmonary fibrosis, granulomatous colitis. Explanation: Menkes kinky hair syndrome is transmitted in an X-linked recessive manner and is caused by a mutation in ATP7A, an ATP-dependent copper tranporter. This defect results in low serum levels of copper. These individuals will have hair abnormalities such as sparse, hypopigmented brittle hair, eyelashes and eyebrows, lax skin, a "cupid's bow" upper lip, CNS progressive deterioration, seizures, skeletal abnormalities and tortuous arteries.The x-linked recessive type of dyskeratosis congenita displays a mutation in: A patient with a white, spongy overgrowth of the buccal mucosa that has passed in an autosomal dominant fashion is most likely related to a mutation in which of the following? Which syndrome is due to a defective secreted mammilian Ly6/uPAR-related protein-1?This is the most common type and it is a mutation in dyskerin This description is most likely a white sponge nevus, an autosomal dominant defect in keratin 4/13. Mal de malada (SLURP 1)Which of the following laboratory test might prove useful in the diagnosis of Fabry disease: A patient has Conradi-Hunermann-Happle syndrome with congenital ichthyosiform erythroderma, ventricular septal defect, and asymmetric limb shortening. The bone finding for the disease is: Which disease can clinically mimic pellagra but is inherited in an autosomal recessive fashion and is due to a defect in the transport of neutral amino acids?Urinary sedimentation rate with polarizing light microscopy Chondrodysplasia punctata The clinical manifestation of Hartnup disease is similar to that of pellagra because the resultant defect in the transport of amino acids leads to low levels of tryptophan. Since tryptophan is required to make nicotinic acid, pts with Hartnup disease manifest the same symptoms as niacin-deficient patients (pellagra).Giant Congenital nevus gene what is low in patients with acrodermatitis enteropathica? DM antibodies in kids for photosensitivity, severe skin diseaseNRAS (somatic mutation) both zinc and alk phos are low!! TIF1 = photosensitivity in kids MDA5 = vasculopathic, severe skin disease, interstitial lung diseaseThe finding of 'maltese crosses' in the urine is characteristic of which of the following conditions? How is acrodermatitis enteropathica inherited? An infant presents with poikiloderma on his face, buttocks, arms and legs. He is also noted to have a hypoplastic thumb and no radius. Yearly ophthalmologic examination is indicated because of the infant is at risk for developing:Fabry disease autosomal recessive Cataracts (Rothmond Thompson)Which of the following hereditary skin disorders is associated with the RAS-ERK-MAPK pathway? A child presents with a white forelock and a well-circumscribed, irregularly shaped milky white patch around his elbow with macules of normal pigmentation within its border. He has one brown iris and one iris that is brown and blue. He also has deafness. All findings have been present since birth. The distance between the inner canthi, between the pupils and between the outer canthi are all normal. What is the most likely underlying mutation? A patient has macular telangiectasias on the oral mucosa, face and acral surfaces. These patients also have epistaxis, melena related to angiomas in the GI tract and AV malformation in other organs. The gene mutation is:Cardio-facio-cutaneous syndrome, Costello, LEOPARD, NF, and Noonan. MITF ALK1What condition is associated with a mutation in CXCR4 In a patient suspected of having multiple endocrine neoplasia type IIb, which lab test would be appropriate? A 5-year-old male is diagnosed with neurofibromatosis type 1 (NF1). NF1 is due to a mutation in the gene which encodes neurofibromin. Which of the following best describes the function of neurofibromin?Whim syndrome Calcitonin Turns ras to its active formWhat is the most common cancer associated with cowden disease? Dental enamel pits are seen in which of the following conditions? AKT1 mutation is seen in which of the following conditions?Breast cancer Tuberous sclerosis Proteus syndromeMain cause of death in patients with dyskeratosis congenita? Painful crises and 'whorled' corneal opacities are seen with which of the following enzyme abnormalities? Retention of primary teeth is a prominent dental finding of which of the following conditions?pancytopenia Alphagalactasidase A (fabry) Hyper IGE syndromehow are epidermal nevus syndromes inherited? What finding is seen on brain imaging of patients with Papillon-Lefevre Syndrome?sporadically Dural calcificationsA 37-year-old woman presents with the fingernail findings pictured. She also has visible deformities of her right elbow that restricts extension. X-rays of the knee and pelvis show absent bilateral patella and the presence of iliac horns; respectively. X-ray of the elbow show bilateral dislocation of the radius. Which of the following is the most appropriate screening test to perform for this patient? 4 year old girl suffers multiple fractures. She also has kyphoscoliosis and blue sclera. An echocardiogram reveals mitral valve prolapse. What type of osteogenesis imperfecta does she most likely have?urinalysis: This patient has nail patella syndrome (NPS), a disorder that presents with the tetrad of fingernail dysplasia, hypoplasia or absence of the patella, presence of iliac horns, and elbow deformities, although ocular, renal, and neurological involvement often exists. Patients are at increased risk of developing kidney disease. There are two patterns of NPS nephropathy, the first is an accelerated age related loss of filtration ability involving the podocytes, the second is the development of nephrotic range proteinuria in childhood or young adulthood that can progress to ESRD. Renal involvement is often diagnosed by the presence of hematuria, proteinuria and microalbuminuria. It can also present with new onset hypertension. Therefore, an annual screening urinalysis should be performed to screen for early indications of kidney involvement. Type 1An infant with doughy, redundant skin and short sparse hairs is likely to show which features on x-ray? Ectopia lentis with downward displacement of the lens is characteristic of: The most common cutaneous neoplasm associated with Muir Torre Syndrome is which of the following?Metaphyseal widening in the long bones (menkes disease) Homocystineruia Sebacceous adenomaYou are consulted on a patient with possible Nethertons Syndrome. Which location of the body would most likely have hairs demonstrating trichorrhexis invaginata? A 5-year-old girl with a history of asthma is brought in by her parents due to concerns of scaling and peeling of her skin that has been intermittent in nature for the past year. Her birth history is unremarkable. She has noted only mild improvement with topical triamcinolone 0.1% cream. Her IgE level at that time was found to be mildly elevated. Examination of the skin reveals the findings pictured distributed on the extremities and face. Her parents also note sparse and brittle hair in the eyebrows and scalp. Which of the following is the underlying mutation in this condition?eyebrow SPINK 5 = nethertonsmultiple clear cell acanthomas can be found with what conditions? the most common eye finding in patients with CMTC is what? Ingestion of lithium and worsen what condition?They can be found in icthiotic conditions such as lamellar ichthyosis Glaucoma, which is also seen in struge webber and NF1 darier diseaseWhat is the defect in loose anagen hair syndrome? Wart under the nail in young child, worry about what? what should be done if suspected What is the pathophysiology of OCA type 1?inner root sheath keratinization subungal exostosis, order X-ray lack of tyrosinaseErythema Multiforme - what are the two forms? what distinguishes these two forms? - most common cause? other causes - drug causes - Treatment? what is of little benefit?Two forms of EM are recognized - EM minor and EM major. Both are characterized by the same type of elementary lesions (targets), but are distinguished by the presence or absence of mucosal involvement and systemic symptoms (fever, Arthralgias) - HSV1 and 2 are the most common causes. mycoplasma pneumonia, group A Streptococcus, and Epstein-Barr virus are other infectious etiologies - Importantly, mycoplasma pneumonia-induced EM is associated with prominent mucositis and absent or sparse cutaneous findings, Drug causes include penicillins, NSAIDS, anticonvulsants, a - In children, important additional triggers to consider include drugs (particularly penicillin), , varicella, vaccines. - For HSV-induced EM, treating with oral antivirals does not affect the clinical course, so antiviral therapy is not indicated. systemic steroids, citing an increased duration of illness and a higher rate of complications.............Recurrent HSV/EM- Acyclovir 5-20mg/kg for 6-12 mon As a rule, antiviral therapy has minimal impact if given after the appearance of the acute episode of EM. In most cases of EM, especially EM minor, symptomatic treatment will suffice. Oral antihistamines for 3 or 4 days may reduce the stinging and burning of the skin. In severe EM with functional impairment, early therapy with systemic corticosteroids (e.g. prednisone [0.5-1 mg/kg/day for 3-5 days] or pulse methylprednisolone [20 mg/kg/day for 3 days])Trichodysplasia Spinulosa - cause? - treatment? infantile perianal papule - where is this lesion found?-Immunosuppressed state (cyclosporine/transplant) with polyomavirus (MCC) -Sample tissue and run PCR for virus. -Tx- decrease immunosuppression, topical cidofovir (BLOCKS VIRAL REPLICATION) -Infantile perianal pyramidal protrusion: tissue midline anterior to anus (gooch) - Resolve on own, resolve constipation, look out for LSC as constipation is presenting symptom,Goldenhar syndrome Patient with onychodystrophy and sparse hair / conical teeth, what is likely complication What is the pathophysiology of x-linked ichthyosisGoldenhar syndrome(oculo-auriculovertebral dysplasia) - 1st/2nd branchial arches->ears vertabae, heart, eyes, skin tags, microtia (tiny deformed ears), accessory tragus.With management, normal life expectancy. Hyperthermia: hypohidrotic ectodermal dysplasia increased cholesterol sulfate which lead to desquamation of stratum corneum vs increased granular layer vs cornoedesmosome retention in the stratum corneumPossible consequences for child with infantile hepatic hemangioma? Strep vs staph toxic shock syndromehigh output cardiac failure, hypothyroidism -Differences b/n staph TSS and Strep TSS · Diffuse macular erythroderma is commonly seen in staphylococcal but not in streptococcal TSS. · Soft tissue infections are rare in staphylococcal TSS but common in streptococcal TSS. · Positive blood cultures are seen in less than 15% of staphylococcal TSS cases but in over 50% of streptococcal TSS. · Mortality rates are approximately 3% in staphylococcal TSS and 30%-60% in streptococcal TSSHow do JAK inhibitors work in alopecia areata Risk Factors for amniotic band syndrome What is the treatment for rockymountain spotted fever in a 6 year old?How do JAK Inhibitors work in Alopecia areata? -blocks JAK1/2/3 ->blocking of STAT signaling pathway->decrease Tcell activation/IFNg/IL15 Maternal risk factors include epidermolysis bullosa, connective tissue disorders, abdominal trauma, uterine malformations, primigravida mothers younger than 25 years, and amniocentesis. doxycycline is recommended in children of all ages if RMSF is suspected. less than 45 kg: 4.4 mg/kg per day in 2 divided doses 7-14 days (at least 3 days after fever)Most common type of MF in kids? Most likely cutaneous finding in lumbosacral area to indicate an underlying spinal dysraphism? What drug can cause hypertrichosis? What kind of xanthomas do you see with a teenager on isotretinoinhypopigmented MF Lipoma Cyclosporine eruptiveEpidermal nevus associations ILVEN associations When Can patients return to school with hand foot and mouth?epidermal nevus associations = musculoskeletal ILVEN associated with thyroiditis and verruciform xanthomas after fever and blisters dried upmutations in the following lesions - spitz , common acquired nevus, congenital melanocytic nevus, melanoma Cause of SSSS What is the mechanism of action for the drug used to treat tinea?HRAS, BRAF, NRAS, BRAF (on sun exposed areas), Exfoliative toxin that works on desmoglein 1 inhibits 14 alpha demethylaseWhat lab abnormality may you see in a collodion baby? Erythema nodosum causes and treatment Best treatment to prolong life in harlequin ichthyosis?hypernatremia secondary to transepidermal water loss -idiopathic>Strep, EBV, meds, malig, IBD, cocci, sarcoid, behcets -TX: RICE, NSAIDS, potassium iodide, steroids. Oral retinoidMechanism of action of acyclovir? Target Dose of isotretinoin? Child with erythema migrans, how do you treat? Biologic that can be used to treat LCHinhibits viral DNA polymerase 120-150mg/kg Amoxicillin for 21 days, doxy for 8 years and older vemurafenib (BRAF inhibitor)what conditions are associated with the following: - enamel pits - peg teeth - anodontia - odontogenic cysts - retention of primary teethEnamel pits are the characteristic dental findings in tuberous sclerosis. Peg teeth are found in incontinentia pigmenti and anhidrotic ectodermal dysplasia. Anodontia is found in hypomelanosis of ito and incontinentia pigmenti. Odontogenic cysts are seen in Gorlin syndrome, and retention of primary teeth is characteristic of Job syndrome.- Unna Thost ppk inheritance, gene - Howel evans gene - Vohwinkel syndrome gene - haim munk defect - papillon lefevre defectKeratin 1/9 are defective in Unna-Vorner/Thost palmoplantar keratoderma, an AD diffuse symmetric non-transgradiens PPK. TOC gene is defective in Howel-Evans syndrome, an AD PPK associated with focal, pressure-related, non-transgradiens PPK and esophageal cancer/oral leukoplakia. Loricrin mutations are seen in Vohwinkel syndrome variant and symmetric progressive erythrokeratodermia. Cathepsin C defects are seen in Haim-Munk syndrome (PPK+periodontitis+acroosteolysis+onychogryphosis) and Papillon-Lefevre syndrome (sharply demarcated transgradiens, stocking-glove PPK+periodontitis+dural calcifications and choroids attachments).Which genetic defect could explain cutaneous findings in addition to abnormal immunoglobulin levels, recurrent respiratory infections, hypogonadism, and an increased risk of leukemia and lymphoma? Primary pigmented nodular adrenocortical disease and psammomatous melanotic schwannomas are characteristic of which of the following syndromes? Associations for elastosis serpiginosaREQL3, bloom syndrome Carney complex elastosis perforans serpiginosa and is associated with medications and certain systemic conditions. It is associated with the medicine penicillamine, Marfan's syndrome, Down's, osteogenesis imperfecta, and Ehler's Danlos syndrome, acrogeria, cutis laxa, pxe and progeria, rothmund thompson and scleroderma. It is often self limiting.Beckwith-Wiedemann syndrome is characterized by which triad Sturge webber is associated with what eye abnormality? Variants of xeroderma pigmentosum are due to all of the following defects except: - nucleotide excision repair - endonuclease -thymidine kinase - post replication repair - Helicaseexopthalmos, macroglossia, and gigantism glaucoma Thymidine kinaseMost common lab abnormality in wiskott aldrich syndrome? What is the best screening test for hemochromotosis? Sphenoid wing dysplasia is seen in what condition? Sweetsthrombocytopenia Ferritin NF1A patient with myotonic dystrophy and multiple skin lesions most likely has activating mutations in which of the following? cutis vertices gyrata is associated with what condition?beta-catenin.....the skin lesions are pilomatricomas pachydermoperiostosisMost common mutation in blue neviThis question is asking for the examinee to recognize that this is a blue nevus based on the clinical and dermatoscopic morphology. The most common mutaion in blue nevi is GNAQ, as it is found in 50-85% of blue nevi.BRAF is the most common mutation, seen in 65-80% of common acquired nevi. BRAF is often mutated in melanomas on sites of intermittent sun exposure. NRAS is mutated in 80% of congenital nevi and a minority of acquired nevi. HRAS is seen in 15% of Spitz Nevi. PTEN is mutated in 60% of nevi of patients with xeroderma pigmentosum.A 4-year-old girl presents with a fever that lasted about 5 days. Afterward, she developed the skin findings pictures along with injection of the conjunctiva, strawberry tongue, and a widespread morbilliform eruption of erythematous macules and papules. Mom reports swelling of the hands and feet that has improved, with desquamation present now. On physical exam you note an enlarged cervical lymph node. What is the most appropriate treatment for this patient?The case describes the classic findings in Kawasaki disease. Treatment of choice is IVIG plus high-dose aspirin. IVIG should be administered as soon as possible, a single dose of 2 g/kg over 8-12 hours is superior to multiple daily doses in preventing coronary artery aneurysms. IVIG decreased the mortality from 2% to 0.3% and the incidence of aneurysm from 25% to 5-10%. Aspirin is recommended at 30-100 mg/kg/day for 14 days or until afebrile for 48-72 hours, then continued at 3-5 mg/kg daily for 6-8 weeks.Supportive care is not appropriate due to the risk of serious complications, like coronary artery aneuryms. Systemic steroids have conflicting results when used in conjunction with IVIG and aspirin, but should not be used alone. Acyclovir is used in the treatment herpes simplex virus adn varicella-zoster virus infection.What is the function of the gene which is defective in ataxia-telangiectasia? is medulloblastoma a major or a minor criteria for diagnosing gorlins? how about calcification of the falx? Which palmoplantar keratoderma is characterized by painful focal hyperkeratosis and keratitis?DNA repair protein medulloblastoma is a minor and calcification of the falx is a major Richnar hanhartWhich of followings syndromes characterized by Seborrheic-like or exfoliative dermatitis: list other causes of zinc deficiency aside from being weaned off breast milkLeiners disease as well as in parenteral nutrition use, alcoholism because of poor nutritional intake, malabsorption, IBD, diets high in grains containing phytate which binds zinc, and metabolic stress.what drug can be used to treat lymphangioma circumscriptum? labs for dermatomyositis patient with red hair and freckling have what increased risk for melanom or NMSCrapamycin, (MTOR inhibition) aldolase, CPK (not muscle biopsy) 2-3xWhat can be used to treat DSAP? severe pustular disease in young kid with deficiency in? you diagnose someone with IP next stepcholesterol Il1 or IL36 receptor antagonist eye examyou see a labia of a young girl that is swollen or a swollen penis or lips could be a sign of what disease? what prolongs survival for a harlequin baby what can exacerbate nethertons?crohns disease (non-casseating granulomas), can also be associated with perianal skin tag oral retinoid retinoids, also dont use calcineurin inhibitorsTreatment for en coup desabre? nevus of Ota most commonly associated with what? most common cause of majocchi? is T tonsurans an endotrhix or ectothrix?MTX glaucoma most commonly, then melanoma T rubrum endothrixalopecia areata can be associated with what conditions? nail findings of alopecia areata discuss CD4 vs CD8 for PLEVA and PLC. what is the treatment for these conditionsatopy, vitiligo, autoimmune thryoid disease, lupus, IBD, nail pitting (regular and geometric), trachyonychia PLEVA CD8, PLC CD4. Treatment with NB UVB and oral antibiotics (erythro, azithro)Linear IGA - drug cause - associated antibodies Patients with vulvar lichen sclerosis are at increased risk for what? What are the size cut offs for congenital nevi?- vancomycin o IgA autoantibodies against two related antigens of BPAG2 § LAD-1 (120 kD cleaved portion of BP180 antigen) § LABD97 (97 kD cleaved portion of LAD-1) vulvar SCC · Small (1.5cm), Medium (1.5-19.9cm), Large (>20cm)XLD genodermatoseschild incontinentia pigmenti conradi hunnerman hapli bazex dupri christalLangerhans Cell Histiocytosis Erythema nodosumlangerhands cell histiocytosis - BRAF and MAP2K mutations may play a role o Lytic lesions of bones o Insipidus (diabetes) o Nail involvement o Granulomas (birbeck & eosinophilic) o Exophthalmos o Rash (brown or purple) o S-100 (+) & CD1a and langerin positiveJuvenile DM -is it associated with increased risk of malignancy? - autoantibodies - types - labs- no increased risk o MDA5/CADM-140- a/w ILD in kids o Anti-p155/140- a/w extensive skin dz in kids but no increase in malignancy o Anti-p140 (aka NXP2)- a/w calcinosis & contractures in kids o Classic JDM (Brunsting variant) § MC (90%) § Gradual onset of classic skin & mm dz § Freq calcinosis cutis -> favors site of trauma (fingers/elbows/knees/butt), may ulcerate § Steroid responsive o Vasculopathic/ulcerative JDM (Baker variant) § Rare § Rapid onset of severe muscle disease § Severe vasculitis w/ cutaneous ulcerations, livedo reticularis, severe periungual capillary alterations, mm infarction, GI ulceration § Recalcitrant to steroids, poor prognosis Labs: ana, CK, aldolaseParvovirus B19 is what type of virus? A 3 month-old girl with multiple hemangiomas along her right jaw is most at increased risk for: A child presents with high fever, strawberry tongue, lymphadenopathy, and a polymorphous exanthem. Approximately how many of these patients will have coronary artery aneurysms if left untreated?SS DNA virus Subglotic Hemangioma 25%The muscle that is affected in a patient with a fibromatosis colli is:Fibromatosis colli affects the lower sternocleidomastoid muscle that my arise from birth trauma. These occur in the pediatric patients and can spontaneous remit.Congenital Varicella Syndrome occurs after maternal varicella infection during which stage of pregnancy?First 20 weeksExplanation: Congenital Varicella Syndrome occurs after maternal varicella-zoster virus infection early in pregnancy, most commonly between 8 and 20 weeks gestation. Characteristic findings of affected infants include some or all of the following: intrauterine growth resrtriction, cicatricial skin lesions, ocular defects, limb abnormalities, CNS abnormalities.Newborns born to mothers who develop symptoms of VZV from 5 days before to 2 days after delivery are at risk of neonatal varicella.Coxackie A 16 virus is what type of virus? Most common cause of neonatal purpura fulminans? When do koplik spots appear in relation to the exanthem?ssRNA virus protein C deficiency beforeIndividuals with which of the following syndromes characteristically present with photosensitivity, mental retardation, a "wizened" appearance, "bird-headed" facies, and "Mickey Mouse" ears? Which of the following lesions demonstrates a pseudo-Darier's sign what should be ruled out in a patient you suspect of having acropustulosis of infancycockayne syndrome Smooth muscle hamartoma scabiesWhat is the most common extracutaneous manifestation in PHACES syndrome? Patient with self healing papular mucinosis is associated with what non-cutaneous finding EPS can be associated with which drug?cerebrovascular followed by cardiac Arthralgias penicillamineWhat nail finding can be associated with eczema coxackium or hand foot mouth disease? Name the types of PRP in childrenonychomadesis Type three is the classic childhood variant, type 4 is the atypical childhood variant but the most common type